You are on page 1of 45

SBI Clerk 2021 Prelims Capsule

SBI Clerk Prelims 2020| Memory Based Paper |


Questions & Solutions
REASONING ABILITY
Direction (1-5): Study the following information carefully 6. Statements: Some Banana are Orange.
and answer the questions given below: All Orange are Apple.
Conclusions: I. Some Banana are not Apple.
There are eight employees of a company and all of them are II. Some Orange are Banana.
working on eight different designation of a bank viz.
Chairman, CFO, GM, DGM, AGM, Manager, Junior Manager 7. Statements: Only a few East are North.
and Clerk. All the designations given are to be considered Few North are South.
in a given order (as Chairman is considered as Senior-most All South are West.
and Clerk is considered as the Junior-most). Conclusions: I. All East being North is a possibility.
II. Some East are West.
Only two persons are senior to B. One designation lies
between B and G. The number of persons junior to G is 8. Statements: Only a few Song are Melody.
Only a few Melody are Film.
same as the number of persons senior to C. H is just senior
No Film is Award.
to E, but junior to C. More than four designations lie
Conclusions: I. All Song being Film is a possibility.
between H and F. D is junior to A.
II. Some Melody are not Award.
1. How many persons are junior to H? Direction (9-12): Study the following information
(a) None (b) One (c) More than four carefully and answer the questions given below:
(d) Four (e) Three In a certain code language:
2. Four of the following five are alike in a certain way and “Club house near located” is coded as “ol gp ox ot”
hence they form a group. Which one of the following “both club view near” is coded as “mt ox sq ot”
does not belong to that group? “make located house view” is coded as “nk ol gp sq”
(a) A-F (b) B-A (c) H-D “near club view area” is coded as “ot ox sq tm”
(d) G-A (e) E-H 9. What is the code for “both” in the given code language?
(a) ox (b) sq (c) mt
3. How many designation gaps are between A and D?
(d) ot (e) Either (b) or (c)
(a) More than three (b) Two (c) Three
(d) One (e) None 10. What is the code for “house” in the given code
language?
4. Who among the following is just senior to B? (a) ol (b) ox (c) gp
(a) A (b) D (c) C (d) Either (a) or (c) (e) None of these
(d) E (e) None of these
11. The code “nk” is stands for?
5. Who among the following is AGM? (a) make (b) both (c) area
(a) F (b) B (c) G (d) club (e) None of these
(d) D (e) None of these
12. What may be the possible code for “both house” in the
Direction (6-8): In each of the questions below are given given code language?
some statements followed by some conclusions. You have (a) gp mt (b) mt nk (c) mt sq
to take the given statements to be true even if they seem to (d) ox mt (e) gp sq
be at variance with commonly known facts. Read all the
Directions (13-17): Study the following information
conclusions and then decide which of the given conclusions
carefully and answer the questions given below:
logically follows from the given statements disregarding There are eight persons A, B, C, D, E, F, G and H sitting
commonly known facts. around a circular table facing towards the centre of the
(a) If only conclusion I follows. table but not necessarily in the same order.
(b) If only conclusion II follows. B sits third to the right of A. Only one person sits between
(c) If either conclusion I or II follows. B and D. E faces C who is not an immediate neighbour of B.
(d) If neither conclusion I nor II follows. G sits third to the left of F. A is not an immediate neighbour
(e) If both conclusions I and II follow. of F.

13 www.bankersadda.com | www.sscadda.com | www.careerpower.in | Adda247 App


SBI Clerk 2021 Prelims Capsule
13. Who among the following sits to the immediate left of 22. If the position of first and third digits of each of the
C? numbers are interchanged, then which among the
(a) G (b) D (c) F following is the highest number?
(d) H (e) None of these (a) 258 (b) 469 (c) 189
(d) 427 (e) 625
14. Who among the following sits to the opposite of A?
(a) H (b) G (c) F 23. If all the digits in the number are arranged in the
(d) D (e) None of these descending order within the number from left to right,
then which among the following will be the lowest
15. How many persons sit between H and G when counted
number after rearrangement?
from the left of G?
(a) 427 (b) 189 (c) 258
(a) Two (b) One (c) Three
(d) 625 (e) 469
(d) Four (e) None of these
24. What is the product of 3rd digit of 2nd lowest number
16. If all the persons sit around the circle in clockwise
and 1st digit of 2nd highest number?
direction by their names according to English
(a) 32 (b) 72 (c) 24
alphabetical order starting from A, then how many
(d) 36 (e) None of these
persons remain unchanged except A?
(a) Four (b) One (c) Three 25. If 2 is subtracted from each number then how many
(d) Two (e) None of these numbers thus formed are odd numbers?
(a) One (b) Two (c) Three
17. What is the position of B with respect to H? (d) More than three (e) None
(a) Fourth to the left
(b) Fourth to the right 26. How many such numerals are there in the number
(c) Third to the right ‘645903287’ which will remain at the same position
(d) Both (a) and (c) when arranged in ascending order from left to right?
(e) Both (a) and (b) (a) Three (b) Two (c) One
(d) Four (e) None of these
Direction (18-20): Study the following information
carefully and answer the questions given below: 27. If it is possible to make only one meaningful word with
the 1st, 3rd, 5th and 11th letters of the word
Point E is in 15m north of Point D. Point F is in 20m north ‘INHERITENCE’, which would be the third letter of the
of Point C. Point A is in 35m east of Point F. Point P is in word from the left? If more than one such word can be
25m south of Point A. Point E is in 20m east of Point P. formed give ‘Y’ as the answer. If no such word can be
18. What is the direction of point F with respect to point P? formed, give ‘Z’ as your answer.
(a) North-west (b) North-east (c) South-west (a) Y (b) R (c) I
(d) South-east (e) North (d) E (e) Z

19. Four of the following five are alike in a certain way and Direction (28-32): Study the following information
hence they form a group. Which one of the following carefully and answer the questions given below:
does not belong to that group? Eight persons A, B, C, D, E, F, G and H are buying some
(a) C-P (b) A-E (c) A-C products one after another but not necessarily in the same
(d) P-D (e) F-E order.
20. If point X is 20m south of point C, then what is the total At most two persons are buying products before F. Only
distance between point X and point D? one person is buying between D and F. C is buying just
(a) 40m (b) 35m (c) 55m before H. One person is buying between C and D. A is
(d) 65m (e) 85m buying just before E. B is buying before G and after E.

Directions (21-25): Following questions are based on the 28. How many persons are buying their products after D?
(a) None
five numbers given below, Study the given information and
(b) Two
answer the following questions.
(c) More than three
625 427 189 258 469
(d) One
21. What is the sum of the 3rd digit of second number from (e) None of these
left and 2nd digit of third number from right?
29. Who among the following is buying just after G?
(a) 10 (b) 15 (c) 14
(a) E (b) F (c) B
(d) 12 (e) None of these
(d) C (e) None of these
12 www.bankersadda.com | www.sscadda.com | www.careerpower.in | Adda247 App
SBI Clerk 2021 Prelims Capsule
30. If all the persons are arranged in alphabetical order Directions (33-35): In each of the question, relationships
from left to right starting from A, then find how many between some elements are shown in the statements.
These statements are followed by conclusions numbered I
persons remains at the same position (excluding A)?
and II. Read the statements and give the answer.
(a) One (b) None (c) Two (a) If only conclusion I follows.
(d) Four (e) More than Four (b) If only conclusion II follows.
(c) If either conclusion I or II follows.
31. Who among the following is buying exactly between D (d) If neither conclusion I nor II follows.
and F? (e) If both conclusion I and II follows.
(a) E (b) B (c) A 33. Statements: P > Q ≥ R = S < T = U
(d) H (e) None of these Conclusions: I. Q > S II. S = Q
34. Statements: J < D = L ≤ K ≤ Q ≥ R
32. How many persons are buying between E and C?
Conclusions: I. J < Q II. L ≤ R
(a) Five (b) Four (c) Three
35. Statements: O > P = G > B ≥ X = M ≤ H
(d) None (e) Two
Conclusions: I. G > X II. O > M

Quantitative Aptitude

Directions (36-40): Table given below shows the number 40. Find total number of male students participated from
of male and female participated in an event from five all the five schools together.
different schools (A, B, C, D & E). Study the table carefully (a) 2860 (b) 3150 (c) 2940
and answer the following questions. (d) 3200 (e) 3020

Schools Male Female Direction (41–45): What will come in the place of
A 650 450 question (?) mark in following number series:
B 540 420 41. ?, 100, 150, 375, 1312.5
C 720 500 (a) 100 (b) 200 (c) 150
D 560 450 (d) 400 (e) 50
E 680 320 42. 104, ?, 96, 120, 88, 128
36. Find average number of female participated from (a) 112 (b) 110 (c) 114
(d) 118 (e) 108
school – A, B & D.
(a) 400 (b) 380 (c) 350 43. 15, 8, 9, 15, 32, ?
(d) 440 (e) 450 (a) 66 (b) 99 (c) 80
(d) 82.5 (e) 80.5
37. Total male participated from school – B & D together
are how much more or less than total female 44. 6, 8, 14, 26, 46, ?
participated from school – A & C together? (a) 72 (b) 84 (c) 96
(a) 150 (b) 110 (c) 170 (d) 80 (e) 76
(d) 120 (e) 240 45. 72000, 36000, 12000, 3000, 600, ?
38. Total male participated from school – B & C together (a) 120 (b) 200 (c) 300
(d) 150 (e) 100
are what percent more or less than total female
participated from school – A & D together? 46. 12 men can do a work in 10 days while 10 women can
(a) 20% (b) 60% (c) 50% do the same work in 18 days. In how many days 4 men
(d) 40% (e) 30% & 6 women together can do the same work?
120 180
39. If total male participated from school – F are 40% more (a) 7 days (b) 24 days (c) 13 days
than that of from school – A and ratio of female (d) 15 days (e) 18 days
participated from school – B to that of from school – F 47. A car can cover a distance in 4 hour at speed 60 kmph
is 21:32, then find total students participated from then by what percent should the speed of car be
school – F. increased to cover the same distance in 2.5 hr?
(a) 1420 (b) 1550 (c) 1580 (a) 60% (b) 40% (c) 50%
(d) 1460 (e) 1490 (d) 100% (e) 75%
13 www.bankersadda.com | www.sscadda.com | www.careerpower.in | Adda247 App
SBI Clerk 2021 Prelims Capsule
48. The ratio of the ages of Ram and Rahim 10 years ago 59. A & B entered into a business by investing total capital
was 1 : 3. The ratio of their ages five years hence will of Rs 17000. B withdraws Rs 1500 after 6 months and
be 2 : 3. Then, the ratio of their present ages is : gets Rs 8100 as profit out of total profit of Rs 19500 at
(a) 1 : 2 (b) 3 : 5 (c) 3 : 4 the end of year. Find capital of B after 6 months from
(d) 2 : 5 (e) None of these starting.
(a) Rs 7000 (b) Rs 9500 (c) Rs 7500
49. Two trains of length 140m & 120m are running in same
(d) Rs 6000 (e) Rs 6500
direction on parallel tracks with speeds 132 kmph &
80 kmph respectively. How much time will they take to 60. If length of a rectangle increases by 40% while keeping
cross each other? breadth constant then area of rectangle increased by
(a) 7.09 sec (b) 18 sec (c) 11.7 sec 24 m2 and perimeter of original rectangle is 32 m. find
(d) 4.42 sec (e) Cannot be determined breadth of rectangle.
(a) 8.4 m (b) 10 m (c) 6 m
50. A person sold a book at 20% profit. If he had bought it
(d) 14 m (e) 8 m
at 10% less cost and sold for Rs 90 more then he would
have gained 40% profit. Find cost price of book. Direction (61–70): What will come in the place of (?)
(a) Rs 800 (b) Rs 1600 (c) Rs 1500 mark in following question.
(d) None of these (e) Rs 1200 61. 280 ÷ 4 ÷ 2 = 170 – ?
Direction (51–55): In each question two equations (a) 105 (b) 115 (c) 125
numbered (I) and (II) are given. You have to solve both the (d) 135 (e) 145
equations and mark appropriate answer. ?
62. (√144 + √169 ) × 3 = 5
(a) If x = y or no relation can be established
(b) If x > y (a) 375 (b) 325 (c) 350
(c) If x < y (d) 275 (e) 475
(d) If x ≥ y 63. (12 × 5 ÷ 4) × 8 = ?
(e) If x ≤ y (a) 100 (b) 140 (c) 120
51. I. 𝑥 = √25 II. y3 = 125 (d) 80 (e) 90
64. (120% of 750) ÷ ? = 25
52. I. x2 + 2x – 35 = 0 II. y2 + 15y + 56 = 0
(a) 30 (b) 36 (c) 24
53. I. x2 = 81 II. y2 = 64 (d) 18 (e) 48
54. I. 17x2 – 14x – 83 = - 80 II. y2 = 2y + 35 1 5
65. 8 2 − 4 6 =? −3 12
7

55. I. x2 + 4x – 45 = 0 II. y2 − 13𝑦 + 40 = 0 (a) 3


1
(b) 3
5
(c) 2
7
4 12 12
56. A container contains mixture of milk & water in ratio 5 (d) 7 4
1
(e) 5 3
2

: 3 respectively. If 8 lit milk is added in it then ratio of


milk to water becomes 11 : 5. Find difference between 66. 275 + 64% of 750 = 750 + ?
initial quantity of milk & that of water. (a) 25 (b) 8 (c) 10
(a) 5 lit (b) 38 lit (c) 18 lit (d) 15 (e) 5
(d) 30 lit (e) 10 lit
67. √225 + √81 + 122 =?
57. Rs 6000 when invested at a certain rate at SI for 2 (a) 168 (b) 164 (c) 162
years, it fetches Rs 1200. If same sum is invested at (d) 172 (e) 182
same rate for a year compounded half – yearly then 510
find compound interest. 68. ?
= √324 + 3.25
(a) Rs 615 (b) Rs 600 (c) Rs 1200 (a) 12 (b) 48 (c) 24
(d) Rs 585 (e) Rs 1260 (d) 6 (e) 18

58. A boat can cover 28 km downstream in 42 min. ratio of 69. 12.5% of (120 + ?) = 45
speed of boat in still water to speed of stream is 7 : 3. (a) 160 (b) 180 (c) 360
Find difference between time taken by boat to cover 60 (d) 240 (e) 120
km downstream & 40 km upstream. 70. 572 ÷ 13 × 12 – 16 = (8)?
(a) 2.25 hr (b) 1 hr (c) 1.5 hr (a) 4 (b) 2 (c) 3
(d) 0.4 hr (e) 0.9 hr (d) 5 (e) None of these

14 www.bankersadda.com | www.sscadda.com | www.careerpower.in | Adda247 App


50+ Bank PO | Clerk Previous Year’s Papers 2016 – 2020

1 Adda247 Publications For any detail, mail us at


Publications@adda247.com
50+ Bank PO | Clerk Previous Year’s Papers 2016 – 2020

Mock SBI Clerk Prelims 2019


09
REASONING ABILITY

Direction (1-5): Study the following information carefully 6. Who among the following faces Q?
and answer the questions given below: (a) B (b) D (c) A
Eight persons are sitting around a circular table facing to (d) C (e) None of these
the center but not necessarily in the same order. 7. Who among the following sits at the extreme end of the
Two persons sit between Q and P (either from left or right). row?
R sits immediate to the right of Q. One person sits between (a) E (b) T (c) C
R and S, who faces to T. Q and T are not immediate (d) R (e) P
neighbors of each other. W sits 2nd to the left of V. Three 8. Four of the following five are alike in a certain way and
persons sit between U and V. hence they form a group. Which one of the following
1. Four of the following five are alike in a certain way and does not belong to that group?
hence they form a group. Which one of the following (a) Q (b) A (c) T
does not belong to that group? (d) D (e) P
(a) Q-W (b) P-U (c) S-W
9. Who among the following sits 2nd to the right of P?
(d) V-T (e) Q-P
(a) S (b) R (c) Q
2. Who among the following sits immediate right of U? (d) T (e) None of these
(a) W (b) R (c) T
(d) S (e) V 10. What is the position of A with respect to E?
(a) 3rd to the left
3. The number of persons sit between Q and T , when (b) Immediate to the left
counted to right of Q is same as the number of persons (c) 2nd to the left
sit between W and ___, when counted to the left of ___? (d) 3rd to the right
(a) P (b) S (c) T
(e) 2nd to the right
(d) U (e) None of these
11. In the word ‘PRODUCE’, how many pairs of the letters
4. Who among the following faces R?
have the same number of letters between them in the
(a) U (b) V (c) P
(d) W (e) Q given word as they have in the English alphabet series?
(a) Four (b) Two (c) One
5. Who among the following sits 3rd to the right of S? (d) Three (e) More than four
(a) P (b) U (c) Q
(d) T (e) None of these Direction (12-15): In each of the questions below are
given some statements followed by some conclusions. You
Direction (6-10): Study the following information have to take the given statements to be true even if they
carefully and answer the questions given below: seem to be at variance with commonly known facts. Read
all the conclusions and then decide which of the given
Ten persons are sitting in two parallel rows containing five
conclusions logically follows from the given statements
persons in each row such a way that there is an equal
disregarding commonly known facts.
distance between adjacent persons. In the first row, A, B, C,
D and E are seated and all of them are facing north. In the 12. Statements: All Chocolate are Toffee. No Toffee is
second row, P, Q, R, S and T are seated and all of them are Muffin. Only a few Muffin are Dark.
facing south. Therefore, in the given seating arrangement, Conclusions: I. No Chocolate is Muffin.
each member seated in a row faces another member of the II. Some Toffee can never be Dark.
other row. (a) If only conclusion II follows.
E sits 2nd from one of the extreme end of the row. P faces (b) If only conclusion I follows.
the one who sits 2nd to the right of E. D sits 2nd to the left of (c) If neither conclusion I nor II follows.
B, who does not sit at the extreme end. Two persons sit (d) If either conclusion I or II follows.
between S and Q. R sits immediate left of S. C sits next to B. (e) If both conclusions I and II follow.

2 Adda247 Publications For any detail, mail us at


✦ Join ➺ @bhawna_weekly_quiz_pdf Publications@adda247.com
50+ Bank PO | Clerk Previous Year’s Papers 2016 – 2020

13. Statements: No Party is Gathering. All Summary are 20. As many as persons are buying things before D as
Gathering. Only a few Gathering are Committee. after___?
Conclusions: I. Some Committee can be Summary. (a) E (b) B (c) F
II. Some Party can be Committee. (d) A (e) G
(a) If either conclusion I or II follows.
Direction (21-23): Study the following information
(b) If only conclusion I follows.
carefully and answer the questions given below:
(c) If neither conclusion I nor II follows.
(d) If only conclusion II follows. Point S is 15m west of point R. Point R is 30m south of
(e) If both conclusions I and II follow. point Q. Point P is 20m west of point Q. Point U is 15m
south of point P. Point T is 35 north of point S.
14. Statements: No Market is Home. Only a few Home are
Room. Only a few Room is Vance. 21. If point V is exactly between point Q and R, then how
Conclusions: I. Some Home are Vance. far and in which direction is point U with respect to V?
II. Some Market can never be Room. (a)15m, North-East
(a) If only conclusion I follows. (b) 15m, East
(b) If neither conclusion I nor II follows. (c) 10m, North-west
(c) If either conclusion I or II follows. (d) 20m, West
(d) If only conclusion II follows. (e) 20m, North-East
(e) If both conclusions I and II follow.
22. Four of the following are alike in a certain way, so form
15. Statements: Some Ball are Garden. All Garden are a group. Which of the following does not belong to that
Trade. Only a few Trade are Pump. group?
Conclusions: I. Some Ball are Pump. (a) P, T (b) U, Q (c)R, P
II. Some Garden can be Pump. (d) S, Q (e) U, T
(a) If only conclusion II follows.
(b) If neither conclusion I nor II follows. 23. If point W is in 5m east of point U, then what is the
(c) If either conclusion I or II follows. distance between point W and Point S?
(d) If only conclusion I follows. (a) 5m (b) 15m (c) 25m
(e) If both conclusions I and II follow. (d) 10m (e) 20m

Direction (16-20): Study the following information Direction (24-26): Study the following information
carefully and answer the questions given below: carefully and answer the questions given below:
Seven persons are buying different things. Only one person A certain number of persons are sitting in the row. All of
buy thing between C and A. B is buying thing immediately them are facing towards north. Q sits sixth from the right
before C. Two persons buy things between C and D. C buy of S. T sits forth to left of Q. Only two persons sit between
things after D. E buys thing immediately before D. More Q and P. R sits forth to the left of S. U sits between S and T.
than three persons buy things between E and F. Only three V sits second to the right of U. W is third from any of the
persons buy things between D and G. end. S is eight from the left end of the row. Six persons sit
16. How many persons buy things between A and F? between W and V.
(a) One (b) Two (c) Three 24. How many numbers of persons could sit in the row?
(d) Four (e) More than four (a) 14 (b) 18 (c) 23
17. Who among the following buy thing immediately after (d) 15 (e) 20
F? 25. What is the position of ‘W’ with respect to Q?
(a) D (b) B (c) G (a) Fourth to the right
(d) A (e) No one (b) Fifth to the right
18. Who among the following buy thing immediately (c) Fourth to the left
before G? (d) Eighth to the right
(a) D (b) B (c) C (e) Sixth to the left
(d) A (e) No one 26. If X sits immediate right of V then how many persons
19. How many person buy things before A? sit between X and P?
(a) One (b) Two (c) Three (a) Five (b) Six (c) Four
(d) Four (e)More than four (d) Three (e) None of these

3 Adda247 Publications For any detail, mail us at


Publications@adda247.com
50+ Bank PO | Clerk Previous Year’s Papers 2016 – 2020

27. If it is possible to make only one meaningful word with A@34%ENM$86&LDS#986QYZ17%ROG


the 2nd, 4th, 6th and 7th letters of the word @2IB2U&
‘UNILATERAL’ which would be the second letter of the
word from the right end? If more than one such word 31. Which of the following element is sixth to the left of the
can be formed give ‘Y’ as the answer. If no such word fourteenth from the left end of the given arrangement?
can be formed, give ‘Z’ as your answer. (a) 6 (b) % (c) $
(a) Y (b) N (c) L (d) M (e) None of these
(d) T (e) Z
32. If all the symbols are dropped from the series, which
Directions (28-30): In each of the question, relationships element will be twelfth from the right end?
between some elements are shown in the statements. (a) 9 (b) Q (c) R
These statements are followed by conclusions numbered I
(d) Y (e) None of these
and II. Read the statements and give the answer.
(a) If only conclusion I follows. 33. How many such numbers are there in the given series
(b) If only conclusion II follows. which are immediately preceded by a symbol and
(c) If either conclusion I or II follows. followed by a letter?
(d) If neither conclusion I nor II follows. (a) None (b) One (c) Two
(e) If both conclusions I and II follow. (d) Three (e) Four
28. Statements: A ≥ B ≥ C = D > E ≤ F < G 34. How many such letters are there in the given series
Conclusions: I. E < B II. G > E which are immediately preceded by number and
29. Statements: P ≤ R < T = U; Q ≥ T ≤ S ≥ V immediately followed by a symbol?
Conclusions: I. Q > P II. V < R (a) One (b) Two (c) Three
(d) More than three (e) None of these
30. Statements: L > M = O ≥ P; N ≤ M ≥ S ≥ T
Conclusions: I. T ≤ P II. N < L 35. Find the odd one out?
Directions (31-35): Study the following sequence and (a) N64 (b) D86 (c) Y%8
answer the given questions. (d) R27 (e) 8EL

Quantitative Aptitude

Directions (36-40): What will come in the place of (a) 75 (b) 155 (c) 125
question mark (?) in the following number series: (d) 175 (e) 165
36. 11, ?, 16, 21, 29, 41 39. ?, 100, 150, 375, 1312.5
(a) 12 (b) 14 (c) 15 (a) 50 (b) 100 (c) 75
(d) 13 (e) 11 (d) 25 (e) 200
37. 1800, ?, 60, 15, 5, 2.5 40. 0, 6, 24, 60, ?, 210
(a) 300 (b) 600 (c) 120
(a) 130 (b) 170 (c) 90
(d) 240 (e) 360
(d) 120 (e) 150
38. 4, 3, 4, 9, 32, ?
Directions (41-45): Study the bar chart given below and answer the following questions.
Bar chart shows the number of books read by 4 different persons (A, B, C & D) in 2005 and 2006.

100
90
80
70
60
50 2005
40 2006
30
20
10
0
A B C D

4 Adda247 Publications For any detail, mail us at


✦ Join ➺ @bhawna_weekly_quiz_pdf Publications@adda247.com
50+ Bank PO | Clerk Previous Year’s Papers 2016 – 2020

41. Find average number of books read by A, C & D in 2005. 54. 35% of 150 × 16 = ? – 22
(a) 64 (b) 70 (c) 75 (a) 865 (b) 932 (c) 864
(d) 60 (e) 56 (d) 862 (e) None of these
55. (3080 + 6160) ÷ ? = 330
42. Find ratio of books read by B & C together in 2005 to
(a) 26 (b) 22 (c) 28
books read by A & D together in 2006.
(d) 29 (e) 18
(a) 15 : 16 (b) 5 : 6 (c) 1 : 5
(d) 4 : 7 (e) 2 : 3 56. Difference of the compound interest received in first
year and second year at 20% per annum at CI is Rs
43. Books read by A & D together in 2005 are what percent
1200 then find the sum?
more than books read by C in 2006?
2 1 2 (a) Rs 25,000 (b) Rs 36,000 (c) Rs 35,000
(a) 46 3 % (b) 54 3 % (c) 25 3 % (d) Rs 24,000 (e) Rs 30,000
1 2
(d) 33 3 % (e) 66 3 % 57. Find the total distance covered by boat in each
44. Books read by A & C together in 2005 are how much upstream and downstream in 7 hours if the speed of
more or less than books read by B & D together in boat in still water and speed of current is 21 km/h and
2006? 3 km/h respectively?
(a) 24 (b) 14 (c) 18 (a) 280 𝑘𝑚 (b) 294 𝑘𝑚 (c) 315 𝑘𝑚
(d) 22 (e) 28 (d) 301 𝑘𝑚 (e) 322 𝑘𝑚

45. Books read by B & C together in 2006 are what percent 58. Ratio of income of A to that of B is 5:9. If expenditure of
3 4
of books read by B in 2005? A is th of his income and expenditure of B is th of his
8 9
(a) 100% (b) 120% (c) 250% income and sum of their saving is Rs 1950 then find the
(d) 200% (e) 160% difference between their income?
Directions (46-55): What will come in place of (?) (a) Rs 900 (b) Rs 1000 (c) Rs 880
question mark in the following questions? (d) Rs 960 (e) Rs 920
17.28÷? 59. A alone can do a work in 12 days while A and B
46. = 200 together can do that work in 7.5 days. Find the time
3.6 × 0.2
(a) 120 (b) 1.20 (c) 12 taken by C alone to do that work if C takes 3 days more
(d) 0.12 (e) None of these than that of B alone to do that work?
47. 486 ÷ ? × 7392 ÷ 66 = 1008 (a) 33 days (b) 30 days (c) 23 days
(a) 54 (b) 55 (c) 52 (d) 27 days (e) 28 days
(d) 53 (e) 51 60. Ratio of base and perpendicular side of a right-angled
2 1 triangle is 3:4 and its base is equal to the side of a
48. 14 % of 4200 ÷ √576 = (? ) 2
square having area 81 cm2. Find the perimeter of the
7
(a) 125 (b) 225 (c) 25 triangle?
(d) 5 (e) 625 (a) 30 cm (b) 36 cm (c) 33 cm
2 5 3 (d) 42 cm (e) 40 cm
49. 7 × 6 × 8 ×? = 90
(a) 1208 (b) 1108 (c) 1008 Directions (61-65): In each of these questions, two
(d) 1128 (e) 1348 equations (I) and (II) are given. You have to solve both the
equations and give answer
50. (0.05 × 6.25) ÷ 2.5 =? (a) if x>y
(a) 12.55 (b) 0.125 (c) 0.115 (b) if x≥y
(d) 1.25 (e) None of these (c) if x<y
51. 1496 ÷ 17 = ?% of 220 (d) if x ≤y
(a) 25 (b) 40 (c) 50 (e) if x = y or no relation can be established between x and
(d) 75 (e) None of these y.

52. (36% of 180) ÷ 0.4 = ? 61. I. x² - 13x + 40 = 0


(a) 160 (b) 164 (c) 166 II. 2y² - y – 15 = 0
(d) 162 (e) 180 62. I. 5x² + 17x + 6 = 0
53. 0.08% of 55500 – 16.4 = ? II. 2y² + 11y + 12 = 0
(a) 26.6 (b) 28 (c) 29.2 63. I. 7x² - 19x + 10 = 0
(d) 30.4 (e) 32 II. 8y² + 2y – 3 = 0

5 Adda247 Publications For any detail, mail us at


Publications@adda247.com
✦ Join ➺ @bhawna_weekly_quiz_pdf
50+ Bank PO | Clerk Previous Year’s Papers 2016 – 2020

Mock SBI Clerk Prelims 2018


10
REASONING ABILITY

Direction (1 – 3): Read the information carefully and law of L. D is the maternal grandfather of P, who is a male.
answer the questions: Q is the only son of W. W is the grandfather of N and C is
A company ABC printed different number of books in the daughter of N.
different years 1947, 1956, 1987, 1998, 2002 such that 7. How L is related to C?
number of books printed are not same in any year. 66 (a) Mother (b) Son (c) Brother
books were printed in an odd numbered year which is not (d) Father (e) None of these
1947.The number of books printed in 1947 is 10 less than
that printed in 1987. 59 books were printed in an year 8. How is P related to N?
before the year in which 61 books are printed but not (a) Mother (b) Son (c) Brother
immediate before. The number of books printed in 2002 (d) Father (e) None of these
is 2 more than that printed in 1998. Direction (9 – 11): Read the information carefully and
1. How many books were printed in 1947? answer the question:
(a) 56 (b) 66 (c) 63 Point U is 10m north of point Q. Point T is 10m east of
(d) 61 (e) none of these point U. Point S is 15m south of point T. Point P is 20m
2. What is the difference between the number of books south of point Q. Point R is 25m east of point P. Point L is
printed in 1956 and 2002? 15m east of point S. Point M is the midpoint of point U and
(a) 7 (b) 10 (c) 8 P.
(d) 4 (e) none of these 9. What is the distance between point L and R?
3. In how many years the number of books printed are (a) 10m (b) 15m (c) 5m
more than that printed in 1998? (d) 20m (e) 25m
(a) two (b) one (c) none 10. In which direction is point T with respect to P?
(d) three (e) four (a) north-west
4. How many words can be formed from the 1st, 6th, 8th (b) south-west
and 9th letter of a word ‘EMANICIPATE’ by using each (c) south-east
letter once in the word? (d) north-east
(a) two (b) one (c) none (e) none of these
(d) three (e) more than three
11. Which of the following points are inline?
5. If all the letters in the word FIGURES are arranged in (a) P, R, S (b) Q, M, L (c) U, S, T
alphabetical order from left to right in such a way that (d) M, S, L (e) Q, S, L
vowels are arranged first followed by consonants,
then how many letters are there in between U and R Directions (12–16): Read the following information
after the arrangement? carefully and answer the given questions.
(a) two (b) one (c) none Twelve persons sitting in two rows. D, E, F, K, L and M
(d) three (e) four sitting in row-1 and facing north. S, T, U, X, Y and Z sitting
in row-2 and facing south direction. E sits third from one
6. If in the number 39682147, 1 is added to each of the
of the extreme ends. S sits second to the left of the one
digit which is less than five and 1 is subtracted from
who faces E. Only three persons sit between S and T. K sits
each of the digit which is greater than five then how
somewhere right of M. More than three persons sit
many digits are repeating in the number thus formed?
between X and T. F faces one of the immediate neighbours
(a) two (b) one (c) none
of T. Z sits second to the right of Y. The one who faces L
(d) three (e) four
sits third to the left of U. D faces S.
Direction (7 – 8): Read the information given below and
12. Who among the following faces K?
answer the questions.
(a) T (b) S (c) X
All the given members belong to the same the family. J is
(d) Y (e) none of these
the brother of L. J is the only son of R. W is the father-in-

2 Adda247 Publications For any detail, mail us at


Publications@adda247.com
50+ Bank PO | Clerk Previous Year’s Papers 2016 – 2020
13. Who among the following faces the immediate I. There are only two persons sit between Sahil and
neighbor of M? Geeta. More than three persons sit to the left of
(a) Z (b) K (c) D Geeta.
(d) L (e) None of these II. Not more than 8 persons can sit in a row. Ravi sits
second to the left of Sahil. Diya sits 6 places away
14. Four of the following five from a group, which among
from Geeta.
the following does not belong to this group?
(a) T, E (b) U, D (c) Y, L 20. What is the code of ‘right’ in a certain code language?
(d) Z, E (e) Z, K I. The code of ‘every right to reject’ is ‘%47 *32 $53
*95’,
15. Who among the following faces the one who sit to the
II. The code of ‘never reject right turn’ is ‘%62 %47
immediate left of Y?
$51 *32’.
(a) U (b) D (c) X
(d) Z (e) none of these 21. Find the number of boys and number of girls in the
row?
16. How many persons sit between M and D?
I. R sits 18th from left end of the row and Y sits 11th
(a) one (b) two (c) three
from the right end of the row. R and Y interchange
(d) five (e) four
their positions, after interchanging the position
Direction (17): Five people A, B, X, Y, and Z live on five R’s position is 20th from left end.
different floors of a building (such as ground floor II. Total 43 students are in the row and all are facing
numbered as 1 and top is numbered as 6). There are three is same direction.
floors between A and B. X lives one of the floors above Y. Directions (22–26): Study the following arrangement
17. Who among the following lives on third floor? carefully and answer the questions given below:
(a) B (b) A (c) X B5R1@EK4F7©DAM2P3%9HIW8*6UJ$
(d) Z (e) Cannot be determined VQ#

18. Which of the following elements should come in a 22. Which of the following is the fifth to the left of the
place ‘?’ ? seventeenth from the left end of the above
AB3 CE6 FI10 JN15 ? arrangement?
(a) OT20 (b) TO21 (c) OT21 (a) D (b) W (c) *
(d) TS21 (e) None of these (d) 4 (e) None of these

Directions (19–21): Each of the questions below consists 23. Which of the following is exactly in the middle
of a question and two statements numbered I and II given between D and U in the above arrangement?
below it. You have to decide whether the data provided in (a) % (b) H (c) 9
the statement are sufficient to answer the question. Read (d) 3 (e) None of these
both the statements and 24. Four of the following five are alike in a certain way
Given answer: based on their position in the above arrangement and
(a) If the data in statement I alone are sufficient to so form a group. Which is the one that does not
answer the question, while the data in statement II belong to that group?
alone are not sufficient to answer the question. (a) R1E (b) F7D (c) M23
(b) If the data in statement II alone are sufficient to (d) 9HW (e) UJ6
answer the question, while the data in statement I
alone are not sufficient to answer the question. 25. How many such symbols are there in the above
(c) If the data either in statement I alone or in statement arrangement each of which is immediately preceded
II alone are sufficient to answer the question. by a number but not immediately followed by a
(d) If the data even in both statements I and II together consonant?
are not sufficient to answer the question. (a) None (b) One (c) Two
(e) If the data in both statement I and II together are (d) Three (e) More than three
necessary to answer the question. 26. Which of the following is the tenth to the left end of
19. Who sits immediate to the left of Ravi, who is sitting the thirteenth from the right end?
in row. All the persons who are sitting in a row facing (a) F (b) M (c) @
north direction? (d) % (e) 3

3 Adda247 Publications For any detail, mail us at


Publications@adda247.com
50+ Bank PO | Clerk Previous Year’s Papers 2016 – 2020
Direction (27–28): Read the information carefully and 31. If 2 is subtracted from the second digit of all odd
answer the questions: numbers and 2 is added in the first digit of all even
Eight persons A, B, C, D, E, F, G, H are sitting around a numbers, then which number is lowest number after
circular table facing centre. H faces B. Two persons sit the arrangement?
between F and B. E sits 2nd right to D. F sits 2nd right to C, (a) 218 (b) 732 (c) 491
who is one of the immediate neighbors of G. C is not an (d) 929 (e) None of these
immediate neighbor of B. 32. If third digit of highest number is divided by the first
27. Who among the following sits 3rd left to F? digit of lowest number, then what will be the
(a) D (b) C (c) B resultant?
(d) A (e) none of these (a) 4 (b) 6 (c) 4.5
28. Who among the following faces A? (d) 5 (e) None of these
(a) D (b) C (c) B 33. If all the digits in each number are arranged in
(d) G (e) none of these increasing order, then which number will be the
Directions (29-30): Study the following information highest number after the rearrangement?
carefully and answer the given questions. (a) 218 (b) 732 (c) 491
In a certain code language, (d) 563 (e) None of these
‘good key friends’ is coded as ‘xo pe cm’
‘key law found’ is coded as ‘xo og bt’ 34. How many numbers will be there in the given series
‘data key good’ is coded as ‘tu xo pe’ in which addition of first and third digit is greater
than second digit?
29. Which of the following is the code for ‘good’?
(a) xo (b) pe (c) tu (a) One (b) Two (c) Three
(d) cm (e) None of these (d) Four (e) None of these
30. Which of the following word is coded as ‘og’? 35. How many numbers will be there in the given series
(a) law (b) good (c) found in which difference of first and third digit is greater
(d) Either (a) or (c) (e) key
than second digit?
Directions (31-35): The following questions are based (a) One (b) Two (c) Three
on the six three digits numbers given below: (d) Four (e) None of these
563 218 732 491 929

QUANTITATIVE APTITUDE
36. Value A is three times of value B. If value of C is 62.5% 39. In a mixture the ratio of milk and water is 7 : 5. When
of difference between value of A and B, then value C is 56 litres of water is added to this mixture the ratio of
what percent of B?
milk and water in mixture becomes 7 : 3. Find the
(a) 120% (b) 125% (c) 115%
(d) 110% (e) 105% initial quantity of milk and water?
(a) 144 liters (b) 120 liters (c) 116 liters
37. A is 40% more efficient than B and both can complete
a work in 20 days. A and B start work and do it for (d) 140 liters (e) 160 liters
eight day. If remaining work is complete by C in 24
days, then find in how many days C will complete 40. A man invested Rs 8400 at the rate R% on SI for two
work alone? years and gets a total interest of Rs. 2016. If he
(a) 30 days (b) 24 day (c) 20 days invested Rs. 600 more amount on half of the previous
(d) 16 days (e) 40 days rate of interest for two years, then find the interest
38. The length of rectangle is thrice the side of square. get by man?
Difference between perimeter of rectangle and square (a) 960 Rs. (b) 840 Rs. (c) 1080 Rs.
is 40 cm. If breadth of rectangle is 8 cm, then find area (d) 1020 Rs. (e) 1040 Rs.
of square?
Direction(41 – 45): Data given about total number of
(a) 169 cm2 (b) 121 cm2 (c) 196 cm2
students visited three towns, read the data carefully and
(d) 144 cm2 (e) 256 cm2
answer the question:

4 Adda247 Publications For any detail, mail us at


Publications@adda247.com
50+ Bank PO | Clerk Previous Year’s Papers 2016 – 2020
There are 400 students visited three towns A,B and C. Out 48. 567.93 + 455.97 - ? = (27.98)2
of total students 20% visited only town A, students visited (a) 120 (b) 180 (c) 160
only town B are 15% less than that of students visited (d) 220 (e) 240
only town A. 18% of total students visited only town C. 3
√?
Student visited town both A & B but not C are 8 less than 49. 124.99 × + 6.02 × 8.98 = (7.99)2
24.99
students visited only town B, students visited town B & C (a) 6 (b) 8 (c) 4
but not A are half the students visited town A & B but not (d) 5 (e) 7
C. Students visited A & C but not B are half the sum of 4.98
students visited only Town B and only town C together, 50. √?+ 135.98 = 7.89 of 319.98
remaining students visited all the three towns. (a) 3025 (b) 2205 (c) 4096
(d) 4098 (e) 4047
41. Find the total number of students visited at least two
town? 51. A boat travels110 km downstream and come back
(a) 180 (b) 106 (c) 160 upstream in total 32 hours, if speed of stream is
(d) 140 (e) 120
62.5% less than speed of boat in still water then find
42. Total students visited town A is how much more than the speed of stream?
total students visited town C? (a) 5 km/hr (b) 3 km/hr (c) 7 km/hr
(a) 48 (b) 40 (c) 38 (d) 8 km/hr (e) 10 km/hr
(d) 36 (e) 30
52. A and B invested total Rs. 20,000 and start a business.
43. Total students who visited all the three towns is what If ‘B’ gets total profit of Rs. 4500 and ratio between
percent of total students visited town B & C but not A? profit share of B and total profit is 9 : 16, then find
1 2 2
(a) 33 % (b) 14 % (c) 16 % amount invested by A if both A and B invested for
3 7 3
2
(d) 66 3 % (e) 60 3 %
2 same time period?
(a) 8250 Rs. (b) 8060 Rs. (c) 8450 Rs.
44. Total students visited only town C is what percent (d) 8560 Rs. (e) 8750 Rs.
less than total students visited only town A? 53. A 120 meters long train cross a pole in 4.8 sec, then
(a) 10% (b) 12% (c) 15% find in what time train will cross a platform, whose
(d) 8% (e) 18% length is three times of the length of the train?
(a) 19.2 sec (b) 18.6 sec (c) 12.8 sec
45. Find the ratio between total students visited only
(d) 20.4 sec (e) 26.8 sec
town B & only town C together to students visited
only town A? 54. Four years ago, Tina is 18 years younger than Ruchi. If
(a) 4 : 5 (b) 7 : 4 (c) 7 : 2 present total age of Ruchi and Tina is 50 years, then
(d) 7 : 5 (e) 7 : 3 find the ratio between present age of Tina to present
age of Ruchi?
Direction (46 – 50): What will approximate value come
(a) 9 : 17 (b) 17 : 8 (c) 17 : 9
at the place of question mark:
(d) 6 : 17 (e) 8 : 17
11.98 ×15.89 + ∛215.98 ?
46. - =0 55. A shopkeeper marked up an article 40% above its
√288.98 51.98 ×2 −52.98
(a) 485 (b) 459 (c) 559 cost price and gave two successive discounts of 25%
(d) 594 (e) 694 and 10% respectively. If he made a loss of Rs. 352,
1.98 then find the cost price of the article?
47. 12.98 % of 399.98 + √? = 2.93 of 125.98
(a) 5600 Rs. (b) 5000 Rs. (c) 6400 Rs.
(a) 1024 (b) 1296 (c) 961 (d) 6800 Rs. (e) 7200 Rs.
(d) 900 (e) 848

5 Adda247 Publications For any detail, mail us at


Publications@adda247.com
50+ Bank PO | Clerk Previous Year’s Papers 2016 – 2020
Directions(56 – 60): Given below bar graph shows total number of books sold by six book stores P, Q, R, S, T and U on
15 April 2015. Read the data carefully and answer the question:
180
160
140
120
100
80
60
40
20
0
P Q R S T U

56. If ratio between books sold by store R on 15 April, 61. 40% of 50% of 60% of 1200 = ? + 53
2015 and on 15 May, 2015 is 15 : 21. 75% of total (a) 21 (b) 23 (c) 24
books sold on 15 May, 2015 are comic books, then (d) 19 (e) 27
find total books sold on 15 May,2015 which are not 62. (12)2 + (15)2 + ? % of 200 = (22)2 - 5
comic books? (a) 55 (b) 45 (c) 35
(a) 36 (b) 42 (c) 48 (d) 30 (e) 65
(d) 52 (e) 56 63. 145 % of 180 + ? % of 320 = 741
(a) 180 (b) 130 (c) 120
57. Total books sold by store T on 15 April, 2015 is what
(d) 140 (e) 150
percent more than total books sold by store U on 15
April, 2015? 64. 46 ÷ 162 × √16 = ?
(a) 80% (b) 60% (c) 40% (a) 56 (b) 60 (c) 64
(d) 70% (e) 80% (d) 72 (e) 68
58. Find difference between average number of books 65. (√529 − √289) × 12 = 136 − ?2
sold by store R & S on 15 April, 2015 and average (a) 8 (b) 10 (c) 12
number of books sold by store Q & U on 15 April, (d) 16 (e) 20
2015? 66. 45% of 160 + ? % of 180 = 19 % of 900
(a) 40 (b) 25 (c) 35 (a) 50 (b) 55 (c) 65
(d) 56 (e) 60 (d) 45 (e) 35
59. Total books sold by store R on 15 April,2015 is what 2 1 1
67. 13 3 − 7 4 = ? +1 2
percent less than total book sold by store T on 15 11 11 11
April, 2015? (a) 5 12 (b) 7 12 (c) 4 12
1 1 1 11 11
(a) 22 3 % (b) 20 3 % (c) 25 3 % (d) 9 12 (e) 11 12
1 1 18×11+232.5
(d) 28 3 % (e) 33 3 % 𝟔𝟖. =?
17×9−30
60. Total books sold by store T on 15 May, 2015 are 20% (a) 2.4 (b) 4.5 (c) 2.5
more than total books sold by same store on 15 April, (d) 3.5 (e) 6.5
2015, then find total number of books sold by T on 15 69. 8.4% of 300 + 4. 8% of ? = 102
May, 2015? (a) 1200 (b) 800 (c) 600
(a)332 (b) 216 (c) 244 (d) 1800 (e) 1600
(d) 264 (e) 316 70. 734 + ? = 12.8 × 64
Direction(61 – 70) : What will come at the place of (a) 80.2 (b) 85.2 (c) 84.8
question mark: (d) 89.5 (e) 78.2

6 Adda247 Publications For any detail, mail us at


Publications@adda247.com
50+ Bank PO | Clerk Previous Year’s Papers 2016 – 2020

Mock SBI Clerk Prelims 2016


11
REASONING ABILITY

Directions (1-5): To answer these questions study 8. Statements : S > M = Z > T < Q > V
carefully the following arrangement of letters, digits and Conclusions : I. V = S II. Q > M
symbols. 9. Statements : T < U = V  S > P  Q
M 7 Σ 8 L P @ ? 6 N B T Y 3 2 = E $ 4 9 © G H 5. Conclusions : I. S > T II. V  Q
1. How many such letters are there in the arrangement 10. Statements : M  N > R > W, E = J > L  W
each of which is immediately followed by a number? Conclusions : I. E > W II. M > L
(a) Three (b) Four (c) One Directions (11-15): The following questions are based
(d) Two (e) None of these on the five three – digit numbers given below:
2. How many such symbols are there in the arrangement 684 512 437 385 296
each of which is immediately preceded by a number? 11. If 2 is added to the first digit of each of the numbers
(a) Two (b) Three (c) Four how many numbers thus formed will be divisible by
(d) Nil (e) None of these three?
(a) None (b) One (c) Two
3. If all the symbols are deleted from the arrangement, (d) Three (e) None of these
then which of the following will be fourth to the left of
12. If all the digits in each of the numbers are arranged in
the 17th element from the left end? descending order within the number, which of the
(a) 9 (b) E (c) 2 following will be the highest number in the new
(d) Y (e) None of these arrangement of numbers?
(a) 684 (b) 385 (c) 296
4. '78' is related to `P ? 6' and ' ?N' is to`T32'in the same
(d) 437 (e) None of these
way as'2E'is to…….. in the arrangement.
(a)4©H (b)49G (c)4©G 13. What will be the resultant number if the second digit
(d)9GH (e)None of these of the second lowest number is divided by the third
digit of the highest number?
5. If all the numbers are deleted from the arrangement (a) 2 (b) 3 (c) 0
then which of the following will be fifth to the right of (d) 1 (e) 4
the 13th element from the right end? 14. If 1 is added to the first digit and 2 is added to the last
(a) B (b) N (c) Y digit of each of the numbers then which of the
(d) T (e) None of these following numbers will be the second highest
number?
Directions (6-10): In these questions, a relationship (a) 385 (b) 684 (c) 437
between different elements is shown in the statements(s). (d) 296 (e) 512
The statements are followed by two conclusions. Give
15. If in each number the first and the second digits are
answer
interchanged then which will be the highest number?
(a) if only conclusion I is true. (a) 296 (b) 512 (c) 437
(b) if only conclusion II is true. (d) 684 (e) 385
(c) if either conclusion I or II is true. Directions (16-17): Study the following information
(d) if neither conclusion I nor II is true. carefully and answer the questions given below:
(d) if both conclusions I and II are true. P is to the north of Q and S is to the east of P, who is to the
south of W. T is to the west of P.
6. Statements : A > B  C < D, C = E > G
Conclusions : I. D > E II. B > E 16. Who among the following is towards south of W and
north of Q?
7. Statements : P  Q > M  N, Q = S
(a) P (b) T (c) S
Conclusions : I. S > P II. N < S
(d) Q (e) None of these

2 Adda247 Publications For any detail, mail us at


Publications@adda247.com
50+ Bank PO | Clerk Previous Year’s Papers 2016 – 2020
17. W is in which direction with respect to T? 26. Who among the following lives on the topmost floor?
(a) North (b) Northeast (c) Southwest (a) I (b) Q (c) P
(d) West (e) None of these (d) L (e) None of these
Directions (18-22): Study the following information 27. Which of the following combinations is true?
carefully and answer the questions given below: (a) First floor-S (b) Fourth floor-R
Dhondu, Chintu, Titu, Chiku, Sonu, Monu, Bittu and Sonty
(c) Third floor-M (d) Sixth floor-I
are sitting around a circular table facing the center. Sonty
(e) None of these
is third to the right of Titu and second to the left of Sonu.
Chintu is not an immediate neighbor of Sonty and Titu. 28. How many pairs of letters are there in the word (in
Monu is second to the right of Chiku and is an immediate forward direction) APPLICATION, each of which have
neighbor of Titu. Bittu is not the neighbor of Sonu. as many letters between then in the word as they
18. Who among the following is second to the right of have between then in the English alphabet?
Titu? (a) One (b) Two (c) Three
(a) Sonty (b) Bittu (c) Monu (d) Four (e) None of these.
(d) Sonu (e) None of these
29. In a certain coding system, PAPER is written as
19. Who among the following is an immediate neighbor of PERPA and SUBJECT is written as JECTSUB, what
Sonty and Sonu? should be the code for COUNCIL?
(a) Dhondu (b) Chintu (c) Titu (a) NCILCOU (b) LICNOUC (c)NCOUCIL
(d) Bittu (e) None of these (d) NLICUOC (e) NILCCOU
20. In which of the following pairs the second person is
30. In a certain code language ‘lu ja ka hu’ means ‘will you
sitting on the immediate right of the first person?
(a) Dhondu, Sonty (b) Titu, Chiku meet us’, ‘lu ka hu pa means ‘will you sold us’. Then
(c) Bittu, Sonty (d) Sonu, Sonty What is the code of ‘meet’ in this code language?
(e) Monu, Titu (a) ja (b) lu (c) ka
(d) hu (e) cannot be determined
21. Who among the following is second to the left of
Chintu? 31. In a certain code language COMBINE is written as
(a) Titu (b) Sonty (c) Monu XLNYRMV. How will TOWARDS be written in that
(d) Dhondu (e) None of these code language?
22. Who among the following is opposite Chiku? (a) FLDZIWJ (b) GLDZIWH (c) GLEZJWH
(a) Dhondu (b) Bittu (c) Sonty (d) FLEZIWH (e) None of these
(d) Sonu (e) None of these 32. 37 girls are standing in a row facing the school
Directions (23-27): Study the following information building Ayesha is fifteenth from the left end. If she is
carefully to answer the given questions: shifted six places to the right what is her position
Seven neighbours S, P, L, Q, R, M and I live on different from the right end?
floors in the same building having seven floors numbered (a) 16th (b) 21st (c) 20th
one to seven. (The first floor is numbered one, the floor (d) 18 th (e) None of these
above it is numbered two and so on and the topmost floor
is numbered as seven.) 33. X's mother is the mother-in-law of the father of Z. Z is
Three persons live between I and M. M lives on the floor the brother of Y while X is the father of M. How is X
immediately above S, who does not live on an odd- related to Z?
numbered floor. P is neither live on odd number nor (a) Paternal uncle (b) Maternal uncle
topmost floor. I does not live on the first floor. (c) Cousin (d) Grandfather
Two persons live between R and S. Q lives neither on the (e) Brother-in-law
first floor nor on the fourth floor.
34. If A is a brother of B, C is the sister of A, D is the
23. Who lives on the floor just above M? brother of E, E is the daughter of B, F is the father of C.
(a) L (b) P (c) Q than who is the uncle of D?
(d) R (e) None of these
(a) A (b) C (c) B
24. How many persons live between L and P? (d) None of these (e) Can’t be determined
(a) None (b) One (c)Two
(d) Three (e) Can’t be determined 35. A said to B that B’s mother was the mother-in-law of
25. Which of the following pairs live on the first floor and A’s mother. How is A’s mother related to B’s mother?
the topmost floor respectively? (a) Daughter-in-law (b) Mother-in-law
(a) L, Q (b) Q, P (c) I, Q (c) Sister (d) Aunt
(d) L, I (e) Can’t be determined (e) Sister-in-law

3 Adda247 Publications For any detail, mail us at


Publications@adda247.com
50+ Bank PO | Clerk Previous Year’s Papers 2016 – 2020

QUANTITATIVE APTITUDE

Directions (36-40): What should come in the place of 45. Find the difference between the number of various
question mark (?) in the following questions? crimes committed in Bihar and that in Rajasthan.
7 3
(a) 105 (b) 98 (c) 145
36. 5 𝑜𝑓 58 + 8 𝑜𝑓 139.2 =? (d) 139 (e) 104
(a) 133.4 (b) 137.2 (c) 127.8 Directions (46-50): What should come in the place of
(d) 131.6 (e) None of these question mark (?) in the following questions?
37. 12% 𝑜𝑓 555 + 15% 𝑜𝑓 666 =? 46. 4376 + 3209 – 1784 + 97 = 3125 + ?
(a) 166.5 (b) 167.5 (c) 168.5 (a) 2713 (b) 2743 (c) 2773
(d) 169.5 (e) None of these (d) 2793 (e) 2737
38. 84368 + 65466 − 72009 − 13964 =? 47. √? + 14 = √2601
(a) 61481 (b) 62921 (c) 63861 (a) 1521 (b) 1369 (c) 1225
(d) 64241 (e) None of these (d) 961 (e) 1296
39. 337.8 × 331.2 ÷ 335 = 33 × 33? 48. 85% of 420 + ?% of 1080 = 735
(a) 2.8 (b) 3 (c) 3.2 (a) 25 (b) 30 (c) 35
(d) 4 (e) 6 (d) 40 (e) 45
? 324 7 5 1
40. = ? 49. of of of 3024 = ?
529 3 4 9
(a) 404 (b) 408 (c) 410 (a) 920 (b) 940 (c) 960
(d) 414 (e) 416 (d) 980 (e) 840
50. 30% of 1225 – 64% of 555 = ?
Directions (41-45): Study the following table carefully
(a) 10.7 (b) 12.3 (c) 13.4
and answer the given questions:
(d) 17.5 (e) None of these
The number of various crimes, as supplied by
national crime record, reported in different states in 51. How many litres of water should be added to a 30
the year 2012-13. litre mixture of milk and water containing milk and
water in the ratio of 7 : 3 such that the resultant
mixture has 40% water in it?
(a)5 (b)2 (c)3
(d)8 (e) 7
52. The S.I on certain sum of money for 15 months at rate
of 7.5% per annum exceed the S.I on same sum at
12.5% per annum for 8 months by Rs 3250 find sum?
(a)160000 (b)20000 (c)170000
(d) 18000 (e) 312000
53. 4 men and 3 women finish a job in 6 days, and 5 men
41. The total number of various crimes in HP is and 7 women can do the same job in 4 days. How long
(a) 37803 (b) 38903 (c) 37903 will 1 man and 1 woman take to do the work?
(d) 36903 (e) 37003 2 1 1
(a) 22(7) days (b) 25(2) days (c)5 (7) days
42. Find the ratio of Stalking and Assault in UP to Theft 7
(d) 12( ) days (e) None of these
and Criminal Trespass in Haryana. 22
(a) 28 : 51 (b) 21 : 52 (c) 52 : 21 54. A and B started a business with initial investments in
(d) 14 : 55 (e) 55 : 14 the ratio 5 : 7. If after one year their profits were in
the ratio 1 : 2 and the period for A’s investment was 7
43. Find the approximate average of Murder and Theft in
months, B invested the money for
all the eight states together.
(a) 6 months (b) 2 ½ months (c) 10 months
(a) 1141 (b) 1132 (c) 1311 (d) 4 months (e) 7 months
(d) 941 (e) 1021
55. An army lost 10% its men in war, 10% of the
44. The total number of Assaults and Murders together in remaining due to diseases and 10% of the rest were
Bihar is what per cent of the total number of crimes in disabled. Thus, the strength was reduced to 729000
that state? active men. Find the original strength.
(a) 29.82% (b) 39.82% (c) 25% (a) 1000000 (b) 1200000 (c) 1500000
(d) 21.82% (e) 25.5% (d) 1800000 (e) none of these

4 Adda247 Publications For any detail, mail us at


Publications@adda247.com
50+ Bank PO | Clerk Previous Year’s Papers 2016 – 2020
56. What is the difference between the compound (a) 4 (b) 2 (c) 1
interests on Rs. 5000 for 1 years at 4% per annum (d) 3 (e) None of these
compounded yearly and half-yearly?
(a)2 (b)3 (c)4 65. The price of an article is first increased by 20% and
(d)8 (e)none of these later on the price were decreased by 25% due to
reduction in sales. Find the net percentage change in
57. The speeds of John and Max are 30 km/h and 40
final price of Article.
km/h. Initially Max is at a place L and John is at a
place M. The distance between L and M is 650 km. (a) 20% (b) 18% (c) 38%
John started his journey 3 hours earlier than Max to (d) 10% (e) None of these
meet each other. If they meet each other at a place P Directions (66–70): What will come in the place of the
somewhere between L and M, then the distance question mark (?) in the following number series?
between P and M is :
(a) 220 km (b) 250 km (c) 330 km 66. 48, 23, ?, 4.25, 1.125
(d) 320 km (e) None of these (a) 10.5 (b) 10 (c) 2.5
58. The average weight of boys in a class is 30 kg and the (d) 11 (e) None of the above
average weight of girls in the same class is 20 kg. If
the average weight of the whole class is 23.25 kg, 67. 2, 15, 41, 80, 132, ?
what could be the possible strength of boys and girls (a) 197 (b) 150 (c) 178
respectively in the same class? (d) 180 (e) None of the above
(a) 14 and 26 (b) 13 and 27 (c) 17 and 27
(d) 19 and 21 (e) 14 and 27 68. ?, 15, 75, 525, 4725, 51975
59. A profit of 8% is made by selling a shirt after offering (a) 5 (b) 10 (c) 8
a discount of 12%. If the marked price of the shirt is (d) 6 (e) None of the above
Rs.1080, find its cost price
(a) 890 (b) 780 (c) 880 69. 4, 19, 49, ?, 229
(d) 900 (e) none of these (a) 75 (b) 109 (c) 65
4
60. The difference between 5 of a number and 45% of the (d) 169 (e) None of the above
number is 56. What is 65% of the number?
70. 840, ?, 420, 140, 35, 7
(a) 96 (b) 104 (c) 112
(d) 120 (e) None of these (a) 408 (b) 840 (c) 480
(d) 804 (e) None of the above
61. A man can row 24 km upstream and 54 km
downstream in 6 hours. He can also row 36 km
upstream and 48 km downstream in 8 hours. What is
the speed of the man in still water?
(a) 18.75 kmph (b) 19.25 kmph
(c) 17.65 kmph (d) 15.55 kmph
(e)22.75 kmph
62. The numerator of a fraction is decreased by 25% and
the denominator is increased by 250%. If the
6
resultant fraction is 5, what is the original fraction?
22 24 27
(a) 5
(b) 5
(c) 6
28 30
(d) (e)
5 11
63. What would be the area of a rectangle whose area is
equal to the area of a circle of radius 7 cm?
(a) 77 cm2 (b) 154 cm2 (c) 184 cm2
(d) 180 cm2 (e) 150 cm2
64. In a village three people contested for the post of
village Pradhan. Due to their own interest, all the
voters voted and no one vote was invalid. The losing
candidate got 30% votes. What could be the minimum
absolute margin of votes by which the winning
candidate led by the nearest rival, if each candidate
got an integral per cent of votes?

5 Adda247 Publications For any detail, mail us at


Publications@adda247.com
50+ Bank PO | Clerk Previous Year’s Papers 2016 – 2020

Mock SBI Clerk Mains 2019


12
REASONING ABILITY

Directions (1-5): Study the following information and Directions (6-10): Study the following information
answer the questions given below: carefully and answer the given questions.
There are three rows i.e. row 1, row 2 and row 3 Such that
row 2 is in the north of row 3 and row 1 is in the north of A word and number arrangement machine when given an
row 2. There are 4 persons sitting in row 1 and 8 persons input of words and numbers rearranges them following a
are sitting in the row 2 and 4 persons are sitting in the row particular rule. The following is an illustration of input and
3. rearrangement.
Persons sitting in the row 3 faces north. Persons sitting in Input: 42 74 TBEF WRAK 95 NLDG 31 65 KEOR QCOF
the row 1 faces south. First 4 persons sitting from west to Step I: 90 42 74 TBEF NLDG 31 65 KEOR QCOF AKRW
east in row 2 faces north and last four person sitting from Step II: 79 90 42 NLDG 31 65 KEOR QCOF AKRW BEFT
west to east in row 2 faces south.
Step III: 60 79 90 42 NLDG 31 KEOR AKRW BEFT CFOQ
Note: All the persons sitting in the row 1 and row 3 are Step IV: 47 60 79 90 31 KEOR AKRW BEFT CFOQ DGLN
facing the persons sitting in the row 2. Step V: 26 47 60 79 90 AKRW BEFT CFOQ DGLN EKOR
E faces the one who sits second to the right of P. No one sits Step V is the last step of the rearrangement. As per the rules
on the left of E. Only one person sits between P and R. Only followed in the above steps, find out in each of the
two person sits between R and the one who faces F. D sits following questions the appropriate steps for the given
immediate right of F. D does not sits at the end of the row. input.
Q sits second to the right of the one who faces D. A face the Input: IMRE 40 69 RBHI 86 PMCN 25 KDSM 57 VATW
one who sits on the immediate left of Q. G faces S but does
not sits at the end of the row. P is not the immediate 6. How many steps would be needed to complete the
neighbour of G. Only one person sits between K and S. K arrangement?
faces the one who sits third to the right of N. J and M are (a) V (b) VII (c) VI
immediate neighbours. J does not face D. Only two person (d) IV (e) None of these
sits between M and L. More than two persons sits between
B and C, who does not face L. C does not face south. 7. Which step number would be the following output?
45 52 64 91 IMRE 25 ATVW BHIR CMNP DKMS
1. How many persons sit between A and G? (a) II (b) III (c) V
(a) One (b) None (c) Three
(d) IV (e) None of these
(d) Two (e) None of these
2. Who among the following sits second to the right of C? 8. Which of the following would be Step III?
(a) F (b) D (c) G (a) 52 64 91 IMRE 40 25 KDSM ATVW CMNP BHIR
(d) L (e) None of these (b) 52 64 91 IMRE 40 25 KDSM ATVW BHIR CMNP
(c) 64 52 91 IMRE 40 25 KDSM ATVW BHIR CMNP
3. Four of the following five belongs to a group following
(d) 52 91 64 IMRE 40 25 KDSM ATVW BHIR CMNP
a certain pattern find the one that does not belong to
that group. (e) None of these
(a) ML (b) CB (c) RF 9. Which of the following element would be the 4th to the
(d) ED (e) KS left of the one which is 8th from the left in the step IV?
4. Which among the following pairs sits at the ends of the (a) 91 (b) IMRE (c) 64
rows? (d) CMNP (e) None of these
(a) BS (b) EQ (c) KG
(d) RM (e) None of these 10. In Step V, which of the following element would be on
eighth position from the right end?
5. How many persons sit on the right of L?
(a) ATVW (b) BHIR (c) 64
(a) Three (b) One (c) No one
(d) 52 (e) None of these
(d) Four (e) None of these

2 Adda247 Publications For any detail, mail us at


Publications@adda247.com
50+ Bank PO | Clerk Previous Year’s Papers 2016 – 2020

11. In the word ‘HOUSEWARMING’ all consonants are 16. Statements: N * Q % R % O & P @ M
written as their preceding letter and all vowels are Conclusions: I. N % O II. O & M
written as their following letters. Now all letters are 17. Statements: A # B * C * D & E
arranged in alphabetical order from left to right and all Conclusions: I. A * D II. D & B
the repeated letters are eliminated. Then, how many
such pairs of letters are there, each of which have as 18. Statements: K % J * H % G; H * S % T
many letters between them in the word (in both Conclusions: I. H & K II. T * J
forward and backward direction) as they have 19. Statements: B # D @ F # G * H % C
between them in the English alphabetical series? Conclusions: I. B & G II. G @ B
(a) One (b) Two (c) Four
(d) Three (e) More than four 20. Statements: V % W & K # L; W * P @ M
Conclusions: I. M & L II. V * W
12. If we form the word by 1st, 3rd and 5th letter of
Direction (21-25): Study the following information
‘FORMATION’ and 3rd and 4th letter of ‘WOMEN’ then,
carefully and answer the questions given below:
what will be the 3rd letter from right?
(a) R (b) A (c) M Six persons are sitting in a row. Some of them are facing
(d) E (e) None of these North and Some are facing South. They are of different
ages. Person whose age is even numbered doesn’t sit
Directions (13-15): Study the following information immediate right of the person whose age is even
carefully and answer the questions given below. numbered.
A%B (28)- A is 42m in north of B
A$B (13)- A is 27m in south of B Two persons sit between P and T and one of them sits at
A#B (24)- A is 28 in east of B extreme end. Three persons are sitting between the
A&B (25)- A is 29m in west of B persons whose age is 14 and 42. Person whose age is 15
M#L (10), N%M (11), O&N (13), P$O (31), Q#P (38), R%Q sits 2nd to the left of T. There are as many persons sit
(6) between T and person whose age is 15 as between the
persons whose ages are 15 and 28. Q sits immediate right
13. N is in which direction with respect to Q? of the person whose age is 15. One person sits between S
(a) North (b) West (c) North-west and U and neither of them sit at extreme end. Q is older
(d) East (e) South-east than R. T whose age is even numbered is older than U who
14. If Z is the midpoint of the line formed between M and faces North. Person whose age is 20 sits 3rd to the left of the
person whose age is 19.
R, then what is the distance between L and Z?
(a) 35.5m (b) 34m (c) 36.5m 21. How many persons are sitting between R and the
(d) 37.5m (e) None of these person whose age is 20?
(a) One (b) Three (c) Two
15. What is the distance between L and R?
(d) Four (e) None
(a) 50m (b) 51m (c) 46m
(d) 48m (e) 49m 22. Who among the following sits immediate right of U?
(a) S
Directions (16-20): In the following questions, the (b) Person whose age is 19
symbols %, &, #, * and @ are used with the following (c) T
meaning as illustrated below- (d) Person whose age is 42
‘P#Q’ means ‘P is not greater than Q’ (e) None of these
‘P*Q’ means ‘P is neither equal to nor smaller than Q’
‘P%Q’ means ‘P is not smaller than Q’ 23. What is the position of Q with respect to the 2nd
‘P@Q’ means ‘P is neither smaller than nor greater than Q’ youngest person?
‘P&Q’ means ‘P is neither greater than nor equal to Q’ (a) 2nd to the right (b) Immediate left
(c) Immediate right (d) 2nd to the left
Now in each of the following questions assuming the given (e) None of these
statement to be true, find which of the conclusions given
24. What is the age of the person who sits immediate left
below them is/are definitely true and give your answer
of S?
accordingly.
(a) 14 (b) 15 (c) 28
(a) If only conclusion I follows.
(d) 19 (e) None of these
(b) If only conclusion II follows.
(c) If either conclusion I or II follows. 25. How many persons are facing North?
(d) If neither conclusion I nor II follows. (a) Two (b) Three (c) One
(e) If both conclusions I and II follow. (d) None (e) More than three

3 Adda247 Publications For any detail, mail us at


Publications@adda247.com
50+ Bank PO | Clerk Previous Year’s Papers 2016 – 2020

Direction (26-30): Study the following information and Direction (31): Study the following information carefully
answer the given questions: and answer the questions given below:
In alphabetical series each consonant is assigned a Eight persons are sitting in a row and all are facing North.
different number from 1-6 (for ex- B is coded as 1, C- Only two persons sit to the right of A. Two persons sit
2……….H-6) and again those numbers get repeated (for ex- between A and B. Three persons sit between C and G. E and
J-1, K-2…….so on). And for the codes of Vowels is starts G are immediate neighbors of B. D sits immediate left of E.
from the numeric code of Z i.e. if the code of Z is 2 then the F sits 2nd to the right of H.
code of vowel A is 2 and code of E is 3 and so on till U which
31. How many persons are sitting right of G?
is coded as 6.
(a) Two (b) Three (c) Four
Note: The code of vowels lies in the range of 1 to 7.
(d) Five (e) None of these
Besides the above information, following operations are to
32. Rooftop solar power growth has demonstrated an
be applied for coding the words given in the questions
overall positive trend, But this will need to be scaled
below.
up massively to achieve the national target.
If the two immediate digits are same (in the code) then the Assumption:
digit of the letter having higher place value in the English (I) With ongoing improvements to solar cell
alphabet will be changed to ‘♠’ i.e. If the code of a word is efficiency and battery technology, rooftops will
‘225’ hence the code will be changed to ‘♠25’ and if the only get more attractive in the future.
two immediate digits (in the code) are in the form such that (II) Major solar projects that connect to the grid often
the preceding digit is one more than its succeeding digit face the challenge of land acquisition and
then the higher digit will be changed to ‘⧫’ i.e. If the code of transmission connectivity.
a word is ‘435’ hence the code will be changed to ‘35’. (III) A survey helps determine usable rooftops,
separating them from green spaces, and analyses
26. If the code for the word ‘SE_ _ED’ is coded as the quality of the solar resource.
‘3𝑇3♠3’ then what letters will come in the fill in (a) Only II follows
the blanks to make it a meaningful word? (b) Both II and III follows
(a) RV (b) AL (c) EM (c) Both I and III follows
(d) AD (e) Either (b) or (d) (d) Only I follows
27. Which of the following word will be coded as (e) Both I and II follows
‘34623’? 33. To provide proper education, we need to improve our
I. SMOKED II. STAKED III. STOKED education system. To improve education system, we
(a) Only I (b) Only II (c) Either II or III need good teachers. To provide good teachers, we
(d) Either I or III (e) All of the three again need good education. In India there are some
good institutes too but most of the students opt for a
28. Which of the following combination of the words and
job rather than trying the career in teaching field
codes is/are true?
which requires post-graduation as minimum
(a) SLOWLY - ♠36♠2
eligibility criteria because they do not get a good
(b) BELIEVE- 1344 salary there.
(c) BURNT- 1724 Course of Action-
(d) ADVERB- 3♠41 (I) The Salary of the teachers should be hiked to
(e) All are true encourage them for opting teaching.
29. If the code for the words ‘BEAST MODE _____’ is coded (II) The minimum eligibility criterion to be a teacher
as ‘13♠4 ♠6♠2 4634’ in the given coded pattern, should be graduate rather than post-graduation
and PhD to become a teacher as students can get
then what will be the missing word?
a job even after graduation.
(a) CLUB (b) WORK (c) STUN
(a) Only II follows (b) Only I follows
(d) WORN (e) Either (b) and (d)
(c) Either I or II follows (d) Neither I nor II follows
30. What is the code for ‘NECKLACE’? (e) Both I and II follows
(a) 42♠1353 (b) 42♠♠23 (c) 421♠353
Direction (34-38): Study the following information
(d) 42♠1335 (e) None of these carefully and answer the questions given below:
Six persons are working in a company. Their designations
are Chief Executive Officer (CEO), Chief Managing Director

4 Adda247 Publications For any detail, mail us at


Publications@adda247.com
50+ Bank PO | Clerk Previous Year’s Papers 2016 – 2020

(CMD), Managing Director (MD), General Manager (GM), • Punctuality and sincerity are one of the key points
Chief Finance Officer (CFO), Finance Officer (FO). Sequence which will surely reviewed.
of the posts are as above i.e. the post of Chief Executive ⚫ Hard work and dedication towards work will be
Officer (CEO) is higher than Chief Managing Director applauded.
(CMD), the post of Chief Managing Director (CMD) is higher ⚫ Just to quantify an amount in the salary package of
than Managing Director (MD) and so on the post of Chief employee with no reason will not be entertained.
Finance Officer (CFO) is higher than Finance Officer (FO).
39. Which of the following can be inferred from the above
They have different years of experiences. They get
statement?
different annual salaries.
I. Punching machine of office will be evaluated by
Only two person’s designation is lower than the HR.
designation of the person whose annual salary is 7 lakhs. II. There will be minimum 10% increment for
There are as many posts above the post of the person deserving employee based on their performance.
whose annual salary is 14 lakhs as below the post of the III. Some of employees of company have done a
person who have 7-year experience. Annual salary of CEO tremendous job with complete determination and
is 6 lakhs. There are three posts in between the post of the enthusiasm.
persons whose experience are 4 years and 5 years. Person (a) Only I (b) Both II and III (c) Both I and III
whose annual salary is 14 lakhs have just higher post than (d) Only III (e) None of these
the post of the person who have 6 years’ experience and
40. Which of the following undermines the statement
just lower post than the post of the person who have 8
given by Manager?
years’ experience. Two posts are between the posts of the
I. Increment of 15% has been done for all the
person who have 3 years and 8 years’ experience. Person
employees whose 1 year is completed.
whose annual salary is 5 lakhs have higher post than the
II. Performance chart has been prepared by leaders
person whose annual salary is 9 lakhs and lower post than
of different department based on their
the person whose annual salary is 4 lakhs. Person whose
proficiency.
salary is 9 lakhs doesn’t have 4 years’ experience.
III. Relaxation time for late coming of 240 minutes in
34. Who among the following has lowest salary? a year i.e. of 10 minutes twice in a month is given
(a) Person who is CMD to all employees.
(b) Person who have 6 years’ experience (a) Only II (b) Both II and III (c) Both I and III
(c) Person who is GM (d) Only I (e) None of these
(d) Person who have lowest experience
Directions (41-45): Study the following information and
(e) None of these
answer the questions given below:
35. Person who is GM have how many years’ experience?
There are nine boxes which are kept one above the other
(a) 3 years (b) 4 years (c) 5 years
such that the box which is placed at the bottom most
(d) 8 years (e) None of these
position is numbered 1 and so on till the box which is
36. How many persons are senior than the person whose placed at the top position is numbered 9.
annual salary is 14 lakhs? All the boxes contain different number of pens in it.
(a) None (b) One (c) Two
Note: The number of pens in a box is equal to the multiple
(d) Three (e) More than three
of the place number of the box which is kept immediately
37. Which of the following is the annual salary of the above it i.e. The number of pens in the box which is kept at
person who is CFO? the bottom is equal to the multiple of is 2, 4, 6… and so on
(a) 4 lakhs (b) 5 lakhs (c) 7 lakhs and the number of pens in the box which is placed at the
(d) 9 lakhs (e) None of these top (9th position) is 10, 20, 30 and so on.
38. Which of the following pair of combination is/are Only two boxes are kept between the box having 42 pens
true? and box A. Box I is kept at the odd number position but
(a) CMD - 4 lakhs (b) GM – 7 years (c) MD – 6 lakhs immediately below box F. There is only one box is kept
(d) CFO – 5 lakhs (e) None is true
between box E and box having 25 pens. Box D is kept
Direction (39-40): There are some criteria which will be immediately above the box containing 63 pens. Box E does
considered during appraisal of employee-Statement by not contain 63 pens. The number of pens in Box G is equal
manager of a company.
to the difference between the number of pens in box D and

5 Adda247 Publications For any detail, mail us at


Publications@adda247.com
50+ Bank PO | Clerk Previous Year’s Papers 2016 – 2020

box I. Box B is kept above the box having 8 pens. Box G 47. Statement: Start to think of travelling by train for a
contains 21 less pens than box A contains. Only three boxes holiday. A train journey can give one a better view of
are kept between box D and the box containing 12 pens. places on the way which an air journey cannot give.
Only two boxes are kept between box H and the box having You can walk around whenever you want, meet other
travelers and locals, relax and watch the landscape go
12 pens. More than three boxes are kept between box C and
peacefully by. Train travel is both a beautiful and
box H. The box having pens which is a perfect square of 3
affordable way to see the country, and no trip is more
is kept immediately above box C. The number of pens in scenic than the trip by trains.
box D is equal to the sum of the number of pens in box H
Conclusions:
and the box which is placed at 2nd position.
I. While going for a holiday, people want to enjoy
41. Which among the following box contains 42 pens? the view of the places on the way.
(a) Box D (b) Box H (c) Box B II. People should not travel by air when they are
(d) Box E (e) Box I going for a holiday.
Which of the following can be concluded from the
42. How many boxes are kept above box F? given statement?
(a) Three (b) Two (c) Four (a) Only I follows (b) Only II follows
(d) Five (e) None of these (c) Both I and II follow (d) None follows
(e) Either I or II follows
43. Number of boxes between I and the box having 63
Directions: (48-50): Study the following information
pens is same between the box B and the box?
carefully and answer the questions which follow–
(a) Box G (b) 42 pens (c) 81 pens ‘T @ U’ means ‘T is parent of U (either mother or father)’
(d) Box C (e) Both (a) and (c) ‘T # U’ means ‘T is sister of U’
‘T $ U’ means ‘U is grandchild of T’
44. Box F contains how many pens?
‘T % U’ means ‘T is brother of U’
(a) 25 (b) 42 (c) 63
‘T & U’ means ‘T is the son-in-law of U’
(d) 81 (e) None of these ‘T * U’ means ‘T is the wife of U’
45. Total number of pens in the boxes A, G and I is? ‘T © U’ means ‘U is the mother of T’
(a) 153 (b) 149 (c) 155 There are some members in a family having three
(d) 151 (e) None of these generation. The relation between the different members of
the family are defined as follows.
46. Begusarai, a district in Bihar which has seen the phase U@G#J
of kidnapping, extortion, murder to “The Industrial C&I
city of Bihar”. Nowadays there are 4 major industries F%H*C
in Begusarai. Growth rate of Begusarai is top amongst B@H
all districts of Bihar and 5th among all districts of India G*F%H©I
in year 2017. J©A$D
B$E
Which of the following can be inferred from the
above statement? 48. If D is the only child of F, then how is D related to U?
I. Opportunities of the employment has been (a) Son (b) Grandson
increased in Begusarai. (c) Daughter (d) Grand daughter
(e) Can’t be determined
II. There is no case of murder, kidnapping and
extortion in 2017. 49. If A has only one son, then how is J related to F?
III. People of Begusarai are hardworking and keen to (a) Son (b) Brother-in-law
go forward. (c) Brother (d) Father (e) None of these
IV. Top industrialists of India are desirous to have 50. If D has no sibling and B has no granddaughter, then
their industry in Begusarai. how is E related to C?
(a) Only II (b) Only I and III (c) Only I (a) Daughter (b) Wife (c) Nephew
(d) Only III and IV (e) None of these (d) Son (e) Cannot be determined

6 Adda247 Publications For any detail, mail us at


Publications@adda247.com
50+ Bank PO | Clerk Previous Year’s Papers 2016 – 2020

QUANTITATIVE APTITUDE
Directions (51-56): Study the table given below and answer the following questions. Some data is missing in the table.
Table shows data regarding Rs.20 notes received by 5 different banks during demonetization.
Total no. of
% of notes on which Ratio of notes on which (50% Total value of Rs.20 notes
Banks notes received
100% return is given return : 80% return) is given received by bank (in Rs.)
by bank
PNB 15,000 ------- 5:7 -------
SBI ------- 80% ------- 9,00,000
Axis
10,000 ------- 1:4 -------
Bank
BOI ------- 75% 3:5 -------
BOB 24,000 ------- ------- 4,80,000
Note: 1. ‘Return’ is the amount (face value of note) of notes returned by bank to its customers.
2. Each bank has given minimum of 50% return on all notes received by it.
51. If difference of notes on which 50% return is given and 56. If difference of number of notes on which PNB gave
on which 80% return is given of PNB is 1000, then find 50% return and 80% return is 1500 and number of
number of notes on which 100% return is given by notes on which Axis bank gave 80% return are 1750
PNB are what percent of total notes received by BOB? less than number of notes on which PNB gave 50%
(a) 12.5% (b) 37.5% (c) 50% return, then find total number of notes on which PNB
(d) 25% (e) 62.5% and Axis Bank gave 100% return together.
(a) 12000 (b) 13000 (c) 12500
52. If total number of notes on which BOI gave 100% (d) 14000 (e) 13500
return and 50% return are 13500, then find total
amount received by BOI is how much more or less than 57. A spend 30% of his monthly salary on house rent, 40%
of the remaining salary on clothing and he distributes
total amount received by Axis bank?
his remaining monthly salary among his two
(a) Rs.1,20,000 (b) Rs.1,00,000 (c) Rs.1,50,000
daughters and a son in the ratio 5 : 5 : 4. If difference
(d) Rs.1,60,000 (e) Rs.1,80,000
of A’s monthly expenditure on Clothing and monthly
53. If ratio of notes on which (50% return : 80% return) is amount given by A to his son is Rs.24000, then find A’s
given by BOB is same as ratio of notes on which (50% annual expenditure on house rent.
return : 80% return) is given by Axis bank and number (a) Rs.500000 (b) Rs.540000 (c) Rs.550000
of notes on which BOB gave 100% return are 20% less (d) Rs.560000 (e) Rs.600000
than total notes received by PNB, then find ratio of 58. A can complete a piece of work in 33 days and C is
notes on which BOB gave 80% return to that of on three times more efficient than A. Ratio of efficiency of
which it gave 100% return. B to that of C is 3 : 2. If all three starts working
(a) 2 : 3 (b) 1 : 3 (c) 4 : 5 together, then find in how many days the work will be
(d) 5 : 7 (e) None of the above. completed?
1
54. If total number of notes received by BOI is 40% less (a) 2 days (b) 5 days (c) 3 days
2
1
than total number of notes received by SBI, then find (d) 4 days (e) 4 2 days
average number of Rs.20 notes received by these 5
banks. 59. A, B & C entered into a partnership business. Amount
invested by B is 3 times of amount invested by A and
(a) 24,700 (b) 25,000 (c) 25,400
ratio of amount invested by C to that of B is 1 : 2. After
(d) 24,200 (e) Cannot be determined.
11 months, all 3 withdrew Rs.Y. If ratio of C’s profit
55. If percentage of notes on which 100% return is given share to total profit at the end of the year is 35 : 129,
by PNB is 50%, then find the amount earned by PNB in then find profit sharing ratio of A to that of B at the end
this whole transaction. of the year.
(a) Rs.47,750 (b) Rs.44,500 (c) Rs.41,500 (a) 19 ∶ 75 (b) 41 ∶ 53 (c) 67 : 27
(d) Rs.45,750 (e) Rs.48,750 (d) 23 : 71 (e) 31 ∶ 63

7 Adda247 Publications For any detail, mail us at


Publications@adda247.com
50+ Bank PO | Clerk Previous Year’s Papers 2016 – 2020

60. A vessel contains milk and water in the ratio 3 : 1. 64. If male employees in company – K are 425 more than
When 80l mixture is taken out and completely female employees in B in company – X, then find
replaced by milk, then milk becomes 700% of the average number of female employees in A, C & D in
water in the vessel. Find original quantity of the vessel. company - X are how much less than male employees
(a) 240 lit (b) 280 lit (c) 320 lit in company – K?
(d) 200 lit (e) 160 lit (a) 400 (b) 450 (c) 360
(d) 480 (e) 520
Directions (61-65): Study the pie chart and table given
below and answer the following questions. 65. If ratio of employees who are in (18-40) age group to
employees who are in (40+) age group in A, B, C & D in
Pie chart shows the percentage distribution of total company – X is 13 : 12, 13 : 37, 4 : 1 & 33 : 7
employees of a company – X in 4 different departments (A, respectively, then find ratio of total no. of employees
B, C & D) and table shows the number of male employees of (18 – 40) age group to total no. of employees of
in these departments. (40+) age group employees in company – X.
Total employees = 2000 (a) 2 : 1 (b) 9 : 5 (c) 5 : 4
(d) 3 : 2 (e) None of the above.
D A Directions (66-72): Find the wrong number in the
20% 25% following number series.
66. 8, 10, 20, 70, 320, 1570, 7830
(a) 7830 (b) 10 (c) 8
C B (d) 320 (e) 1570
30% 25%
67. 18, 20 , 43, 133, 537, 2691, 16163
(a) 43 (b) 16163 (c) 133
(d) 537 (e) 2691
Departments Male employees
A 240 68. 124, 140, 108, 156, 92, 172, 78
(a) 108 (b) 124 (c) 78
B 225
(d) 92 (e) 140
C 350
D 160 69. 260, 380, 510, 618, 759, 856, 1008
(a) 759 (b) 1008 (c) 260
61. If 38% of the female employees in C are newly (d) 510 (e) 618
recruited and ratio of male employees to female
70. 267, 343, 610, 953, 1563, 2515, 4079
employees who are newly recruited in C is 3 : 5, then
(a) 4079 (b) 953 (c) 343
find total number of old employees in C is what (d) 267 (e) 2515
percent of total employees in C?
1 2 2 71. 36, 80, 166, 340, 690, 1392, 2798
(a) 83 3 % (b) 74 3 % (c) 65 3 % (a) 690 (b) 36 (c) 340
2 1 (d) 1392 (e) 80
(d) 70 % (e) 78 %
3 3
72. 30, 100, 230, 490, 1010, 2050, 4130
62. If ratio of male employees to female employees in (a) 30 (b) 4130 (c) 1010
company – Y is 4 : 3 and male employees in company (d) 490 (e) 2050
– Y are 300% more than male employees in C of
73. Ratio of present age of A to that of B is 2 : 5, ratio of
company – X, then find total employees in company – present age of B to that of C is 25 : 18 and ratio of
Y are how much more or less than that of in company present age of C to that of D is 12 : 13. If D is 11 years
– X? younger than B, then find present age of A.
(a) 450 (b) 800 (c) 560 (a) 24 years (b) 20 years (c) 28 years
(d) 630 (e) 750 (d) 30 years (e) 18 years

63. If 16% female employees of B left B and joined D, then 74. Perimeter of a right angled triangle is 60m and length
find the percentage change in no. of female employees of hypotenuse of right angled triangle is 25m. If base
in D after female employees of B joined D. of the right angled triangle is the smallest side, then
2 2 1 find length of smallest side.
(a) 25 3 % (b) 34 3 % (c)18 3 %
2 1
(a) 10m (b) 18m (c) 21m
(d) 30 3 % (e) 35 3 % (d) 25m (e) 15m

8 Adda247 Publications For any detail, mail us at


Publications@adda247.com
50+ Bank PO | Clerk Previous Year’s Papers 2016 – 2020

75. Shopkeeper sells two articles – M & N. He marks article 80. Find the volume of cylinder.
– M 20% above its cost price and he gave 5% discount (I) Curved surface area of cylinder is 1760 cm2 and
on it. Cost price of article – N is 20% more than cost total surface area of cylinder is 70% more than its
price of article – M. If shopkeeper sold article – M at curved surface area.
Rs.285 and article – N at 15% profit, then find selling (II) Volume of cylinder is twice of that of cone. Radius
price of article – N. of cylinder and cone is equal and ratio of height of
(a) Rs.345 (b) Rs.230 (c) Rs.460 cylinder to that of cone is 2 : 3. Height of cone is
(d) Rs.414 (e) Rs.322 30 cm.
Directions (76-80): The following questions are Directions (81-85): In the following questions, two
accompanied by two statements (I) and (II). You have to quantities (I) and (II) are given. You have to solve both the
determine which statements(s) is/are quantities and mark the appropriate answer.
sufficient/necessary to answer the questions.
81. Quantity I: In how many ways a committee of 4
(a) Statement (I) alone is sufficient to answer the
members with at least 2 women can be
question but statement (II) alone is not sufficient to
formed from 8 men and 4 women?
answer the question.
Quantity II: How many 3-digit numbers which are
(b) Statement (II) alone is sufficient to answer the
divisible by 3 can be formed from
question but statement (I) alone is not sufficient to
0,1,2,3,4,5,6,7,8,9, such that 3-digit
answer the question.
number always ends with an even
(c) Both the statements taken together are necessary to
number?
answer the question, but neither of the statements
(a) Quantity I < Quantity II
alone is sufficient to answer the question.
(b) Quantity I ≤ Quantity II
(d) Either statement (I) or statement (II) by itself is
(c) Quantity I > Quantity II
sufficient to answer the question.
(d) Quantity I ≥ Quantity II
(e) Statements (I) and (II) taken together are not
(e) Quantity I = Quantity II or no relation.
sufficient to answer the question.
82. Quantity I: A man invested Rs.5900 for 3 years in a
76. Let t be total number of balls in a bag. Balls are of 3
scheme offering R% p.a. at SI and
colors - black, white and red. Find t.
received Rs.3186 as interest after 3
(I) when one ball is drawn then Probability of getting
1 1 2 years. If the man invested Rs.7900 at
a black ball is 6 , a red ball is 6 & a white ball is 3. (R+5)% p.a. at SI for 3 years, then find
(II) If one white ball is lost and a ball is drawn, then interest received by man (in Rs.).
8
probability of not getting a white ball is . Quantity II: A man invested Rs.X at 13% p.a. at CI for
23
2 years and interest received by him
77. Shivam and Deepak invested in a partnership business after 2 years is Rs.2325.96. Find X (in
in the ratio of 4 : 5. Find the profit share of Shivam. Rs.).
(I) Shivam invested Rs.12000 and period of (a) Quantity I < Quantity II
investment of Shivam and Deepak is 10 months (b) Quantity I ≤ Quantity II
and 4 months respectively. (c) Quantity I > Quantity II
(II) Ratio of period of investment of Shivam and (d) Quantity I ≥ Quantity II
Deepak is 5 : 2 and Deepak’s profit share is (e) Quantity I = Quantity II or no relation.
Rs.12000 less than Shivam’s profit share.
83. Quantity I: Ratio of CP to MP of an article is 19 : 30.
78. Calculate the marked price of item? Shopkeeper allowed 24% discount and
(I) Shopkeeper marked the article 80% above its cost earned 20% profit on selling the article.
price and shopkeeper earned Rs.100 profit on the If SP of the article is Rs.912, then find
article. difference between amount of profit
(II) Ratio of marked price and discount allowed on the earned and amount of discount allowed
article is 3 : 1. (in Rs.).
79. Calculate the rate of interest. Quantity II: Shopkeeper marked an article 70%
(I) Pankaj earned Rs.4500 as interest, when he above its cost price and he allowed 40%
invested Rs.6000. discount on it. If shopkeeper sold the
(II) Pankaj invested equal amount at SI and at CI. After article at Rs.183.6, then find sum of
2 years, CI received by Pankaj is Rs.90 more than amount of profit earned and amount of
the SI received by Pankaj. discount allowed(in Rs.).

9 Adda247 Publications For any detail, mail us at


Publications@adda247.com
50+ Bank PO | Clerk Previous Year’s Papers 2016 – 2020

(a) Quantity I < Quantity II 85. Quantity I: B’s present age is 60% more than A’s
(b) Quantity I ≤ Quantity II present age and ratio of present age of
(c) Quantity I > Quantity II
(d) Quantity I ≥ Quantity II B to that of C is 5 : 2. D is 8 years
(e) Quantity I = Quantity II or no relation. younger than B and D’s present age is
84. Quantity I: A boat can cover distance of 480 km twice of that of C. Find average of
each in downstream and in upstream in present age of A, B, C & D (in years).
total 11 hours. If ratio of speed of boat
Quantity II: Present age of R is equal to average of
in still water to that of stream is 11 : 1,
then find speed of boat in still water (in present age of P & Q. 4 years hence, age
km/hr.). of P is twice of age of Q at that time. If R
Quantity II: A boat can cover a distance of 350 km
is 15 years younger than P, then find
in downstream in 3.5 hours and can
cover a distance of 380 km in upstream age of younger person among P, Q & R.
in 5 hours. Find speed of boat in still (a) Quantity I < Quantity II
water (in km/hr.).
(b) Quantity I ≤ Quantity II
(a) Quantity I < Quantity II
(b) Quantity I ≤ Quantity II (c) Quantity I > Quantity II
(c) Quantity I > Quantity II (d) Quantity I ≥ Quantity II
(d) Quantity I ≥ Quantity II
(e) Quantity I = Quantity II or no relation.
(e) Quantity I = Quantity II or no relation.
Directions (86-91): Study the bar chart given below and answer the following questions.
Bar chart shows the percentage of applicants who applied for renewal of passports on 5 different passport centers (A, B,
C, D & E) and percentage of female applicants who applied for renewal of passports out of total applicants who applied for
renewal of passports.
80
60
in %

40
20
0
A
% of applicants whoBapplied for renewal
C D
of passports E

% of female applicants who applied for renewal of passports

Note: Total number of applicants on a particular center = Number of applicants for new passport on that center + Number
of applicants for renewal of passport on that center.
88. If difference between male and female who applied for
86. If total number of applicants in E are 70% of total
number of applicants in A and ratio of male to female renewal of passports from A is 2400, then find number
applicants who applied for new passports in A & E is of applicants who applied for new passport from A.
7 : 3 & 2 : 1 respectively, then find ratio of total female (a) 9000 (b) 6000 (c) 7000
applicants in A to total female applicants in E.
(a) 85 : 91 (b) 90 : 91 (c) 90 : 93 (d) 10000 (e) 8000
(d) 88 : 91 (e) None of the above.
89. If total applicants from B are 5000 less than total
87. If difference between male and female who applied for applicants from E and male applicants who applied for
new passports from C is 800 and ratio of male to
female who applied for new passports from C is 2 : 3, renewal of passports from B is 3600, then find total
then find total female who applied for passports from number of applicants who applied for new passports
C is what percent of total male who applied for from B & E together
passports from C?
2 2 2 (a) 18500 (b) 21500 (c) 15500
(a) 120 % (b) 122 % (c) 125 %
9
2
9
2
9 (d) 19500 (e) 24500
(d) 116 % (e) 130 %
9 9

10 Adda247 Publications For any detail, mail us at


Publications@adda247.com
50+ Bank PO | Clerk Previous Year’s Papers 2016 – 2020

90. If total applicants from C & E together are 30000 and Directions (96-100): Study the passage given below and
female applicants who applied for renewal of answer the following questions.
passports from C are 800 more than that of from E, There are 3 cities – Delhi, Dhaka and Bangkok. Total
distance between Delhi – Dhaka, Dhaka – Bangkok and
then find average number of applicants who applied Delhi – Bangkok is 19800 km. Distance between Dhaka –
for new passports from C & E. Bangkok is 80% of the distance between Delhi – Bangkok
(a) 9000 (b) 8000 (c) 6500 and ratio of distance between Delhi – Dhaka to distance
(d) 4000 (e) 11500 between Bangkok – Dhaka is 27 : 32.
Fares of 3 different flight operators (A, B & C) on these 3
91. If ratio of applicants who applied for new passports
routes (Delhi – Dhaka, Dhaka – Bangkok and Delhi –
from A to that of C is 2 : 3, then find total candidates Bangkok) are –
who applied from C is what percent more than total
Delhi – Dhaka: Fare of C is 25% more than that of B and
candidates who applied from A?
fare of A is Rs.2700 less than that of B. Ratio of fare of A to
(a) 80% (b) 50% (c) 90% that of C is 3 : 5.
(d) 40% (e) 20%
Delhi – Bangkok: Average fare of A & B is Rs.20000 and
92. When 2 cards are drawn randomly from a pack of fare of C is Rs.8000 more than average fare of A & B.
cards, then find the probability of getting at most 1 ace Average fare of B & C is Rs.26000.
card. Dhaka – Bangkok: Fare of B is Rs.16000 and ratio of fare
209 10 215 of A to that of B to that of C is 6 : 5 : 8.
(a) 221 (b) 13 (c) 221
16 220 96. If Veer wants to travel from Delhi – Dhaka and then
(d) (e) Dhaka – Bangkok without changing flight operator,
17 221
then which flight operator would be the cheapest
93. Ayush invested Rs.75000 in a scheme offering R% p.a. option among A, B & C for Veer.
SI for 5 years and Rs.50000 in another scheme offering (a) A (b) B (c) C
12%p.a. CI compounding annually for 2 years. If (d) Either A or B (e) Either A or C
difference in 2nd year CI and 2nd year SI is Rs.2220, 97. If Deepak is travelling from Dhaka – Bangkok and
then find value of R%. Bangkok – Delhi from C, then find the amount paid by
(a) 4% (b) 8% (c) 6% Deepak per km for the entire trip.
(d) 12% (e) 14% (a) Rs. 3.11/km (b) Rs. 3.98/km (c) Rs. 3.42/km
(d) Rs. 3.62/km (e) Rs. 3.01/km
94. Train – Y crosses Train – X while running in same 98. For which of the following trip, the fare will be
direction in 120 seconds and Train – Y crosses Train – maximum?
40 (a) A, Delhi – Bangkok
X in 3
seconds while running in opposite direction. If
(b) C, Delhi – Dhaka
Train – X is running at 120 km/hr, then find speed of (c) B, Dhaka - Bangkok
Train – Y (in km/hr). (d) A, Dhaka – Bangkok
(a) 150 (b) 180 (c) 200 (e) B, Delhi - Dhaka
(d) 160 (e) None of the above. 99. On Dhaka – Bangkok route, which flight operator is
charging lowest fare per km?
95. A two digit number increased by 27 when its digits are
(a) A (b) B (c) C
reversed and square of digit at units place is 33 more (d) Either B or C (e) Cannot be determined.
than the square of the digit at tens place of the original
100. Find average fare of A, B & C on Delhi – Dhaka route.
number respectively. Find the original number. (a) Rs.12000 (b) Rs.11500 (c) Rs.10400
(a) 69 (b) 58 (c) 25 (d) Rs.11200 (e) Rs.10800
(d) 14 (e) None of the above.

ENGLISH LANGUAGE
Directions (101-105): Read the passage carefully and some, those seemingly rare individuals who do not express
answer the questions based on the passage. an enthusiasm for music are viewed with deep suspicion,
like those who don’t eat chocolate, who are unmoved by
Enjoyed by humans since prehistoric times, music has been
Shakespearean drama, or who fail to express delight over
variously described as the food of love, a shorthand for
an Ashes victory. Yet recently the assumption that the
emotion, and the universal language of humankind. For
11 Adda247 Publications For any detail, mail us at
Publications@adda247.com
50+ Bank PO | Clerk Previous Year’s Papers 2016 – 2020

Mock SBI Clerk Mains 2016


14
REASONING ABILITY

Directions (1-5): Study the following information carefully • Only two people sit between M and L. L is not an
and answer the questions which follow- immediate neighbor of O.
Eight persons S, T, U, V, W, X, Y and Z live on 8 different • N sits second to left of E.
floors in a building the ground floor is numbered 1 then
6. What is the position of L with respect to G?
numbered 2 till numbered 8. They like different colours i.e.,
(a) Third to the right (b) To the immediate left
red, pink, orange, blue, grey, green, yellow and purple.
(c) Second to the right (d) Fourth to the right
There are four floors between W’s floor and S’s floor. W
(e) Second to the left
lives on an odd numbered floor. S like Yellow colur. There
are two floors between T’s and W’s floor. The one who likes 7. Based on the given arrangement, which of the
Grey colour lives on immediate above the floor on which S following is true with respect to N?
lives. U lives immediate above Y. The one who likes red (a) Only three persons sit between N and O.
colour lives on floor numbered one. There is no floor (b) None of the given options is true.
between T’s floor and Z’s floor. V does not live immediate (c) Only one person sits to the right of N.
above or below to W’s floor. Only one person lives between (d) E sits to immediate right of N.
the persons who like yellow and purple colours. The one (e) Both L and F are immediate neighbours of N
who likes pink colour lives immediate above the floor on 8. Who amongst the following people represents the
which the person who likes Grey colour. Z likes blue colour person seated at the extreme left end of the line?
but does not live below to T. X likes Green colour. (a) E (b) N (c) L
1. On which floor Z lives? (d) F (e) H
(a) Third (b) Fourth (c) Fifth 9. How many persons are seated between O and E?
(d) Sixth (e) Seventh (a) Two (b) Three (c) None
2. How many floors are below the floor on which Y lives? (d) Four (e) One
(a) Three (b) Four (c) Five 10. Who amongst the following sit exactly between M and
(d) Six (e) Seven L?
3. Who among the following lives on the second floor? (a) E, N (b) F, O (c) F, G
(a) X (b) V (c) Y (d) E, G (e) H, N
(d) U (e) S Directions (11-15): In each question below is given a group
4. Which colour is liked by V? of letters followed by four combinations of digits/symbols
(a) Orange (b) Pink (c) Grey numbered (1), (2), (3) and (4). You have to find out which
(d) Red (e) Purple of the combinations correctly represents the group of
letters based on the following coding system and mark the
5. Which of the following is true with respect to T? number of that combination as the answer. If none of the
(a) T likes Orange colour and lives on 4th floor? four combinations correctly represents the group of
(b) T like Pink colour and lives on 8th floor letters, mark 5), ie ‘None of these’, as the answer.
(c) T like Purple colour and lives on 4th floor
(d) T like Grey colour and lives on 7th floor
(e) T like Red colour and lives on 8th floor
Conditions:
Direction (6-10): Study the following information to
1) If the first letter is a consonants and the last letter is a
answer the given question. vowel their codes are to be interchanged.
• Eight friends E, F, G, H, L, M, N and O are seated in a 2) If both the first and the last letters is Vowel both are to
straight line, facing north, but not necessarily in the be coded as +.
same order. 3) If both the first and the last letters is Consonant and in
between there are two or more vowels are to be coded
• O sits at the extreme right end of the line. Only four
as 8.
people sit between O and G. 4) If the first letter is vowel and the last letter is a
• Both F and M are immediate neighbours of G. consonant, both are to be coded as the code for the
consonant.
2 Adda247 Publications For any detail, mail us at
Publications@adda247.com
50+ Bank PO | Clerk Previous Year’s Papers 2016 – 2020

11. EFHKLB 21. Statements: B © T, T  M, M % F


(a) @7963* (b) *7963* (c) @7963@ Conclusions: I. B © M II. B © F
(d) *7S63@ (e) none of these.
22. Statements: M  R, R % T, T $ K
12. FTAMIR Conclusions: I. K @ M II. K © M
(a) 7$4812 (b) 7$1842 (c) 7$8882
23. Statements: W © D, D @ H, H  N
(d) 7$189% (e) none of these.
Conclusions: I. N $ D II. W © N
13. DKPRTB
24. Statements: W @ D, D $ R, R © K
(a) *652$% (b) %652$% (c) +652$+
Conclusions: I. R  W II. R % W
(d) *652$* (e) none of these
25. Statements: F $ J, J % V, V © N
14. UPJTRA
Conclusions: I. N $ F II. N % J
(a) 45#$2© (b) ©5#$24 (c) ©5#$2©
(d) +5#$2+ (e) none of these Directions (26-30) : Each of the questions below consists of
a question and two statements numbered I and II. You have
15. HLEKBI
to decide whether the data provided in the statements are
(a) 13@6*9 (b) 93@6*1 (c) 13@6*1
sufficient to answer the question. Read both the statements
(d) 93@6*9 (e) None of these
and answer–
Directions (16-20): Study the following information and (a) If the data in statements I alone is sufficient to answer
answer the questions given. the question while the data in statement II are not
In a certain code language ‘lu ja ka hu’ means ‘will you meet sufficient to answer the question.
us’, ‘fa ka la ju’ means ‘will today maximum temperature’, (b) If the data in statement II alone is sufficient to answer
‘la fu ja ju’ means ‘meet today the temperature’ and ‘ju lu the question while the data in statement I are not
na fu’ means ‘temperature of the us’. then sufficient to answer the question.
(c) If the data in either statement I alone or statement II
16. What is the code of ‘today’ in this code language?
alone are sufficient to answer the question.
(a) ju (b) la (c) fa
(d) If the data even in both statement I and statement II
(d) ka (e) cannot be determined
together are not sufficient to answer the question.
17. What is the code of ‘you’ in this code language? (e) If the data in both statement I and statement II together
(a) hu (b) lu (c) ka are necessarily to answer the question.
(d) ja (e) cannot be determined
26. Among 6 persons U, V, W, X, Y and Z, who are sitting
18. What is the code of ‘you of maximum’? around circle facing to the centre. Who sits immediate
(a) ha hu fu (b) fa hu na (c) fu lu na right of W?
(d) hu fa la (e) cannot be determined Statements:
19. What is the code of ‘us’ in this code language? I. X sits second to the left of Y and opposite to U but not
(a) hu (b) lu (c) ja near to W.
(d) ka (e) cannot be determined II. Only one person is seated between U and V. Y is
seated near to Z.
20. What is the code of ‘meet’ in this code language? 27. How P is related to R?
(a) hu (b) lu (c) ja Statements:
(d) ka (e) cannot be determined I. Q is father-in-law of R, who is mother of S. P is
Directions (21-25): In the following questions, the symbols grandmother of S.
@, ©, %, $ and  are used the following meaning as II. T is father of P, who is son-in-law of U. U has only one
illustrated below: grand-daughter R, who is also a grand-daughter of T.
‘A©B’ means ‘A is smaller than Q’. 28. Who among P, Q, R, S, T, and U is the tallest?
‘A@B’ means ‘A is either smaller than or equal to Q’. Statements:
‘A%B’ means ‘A is greater than Q’. I. S and R are taller than P but shorter than U. Q is taller
‘A $ B’ means ‘A is either greater than or equal to Q’. than T.
‘AB’ means ‘A is equal to Q’. II. U is shorter than T but taller than Q and S. while P is
Now in each of these questions assuming the given not tallest.
statements to be true, find which of the two conclusions I 29. In which direction is Q from of P.
and II given below them is/are definitely true? Give answer Statements:
(a) if only Conclusion I is true. I. Q is towards north of O, which is towards the east of
(b) if only Conclusion II is true. P.
(c) if either Conclusions I or II is true. II. R is towards east of M and P is towards the west of
(d) if neither Conclusions I nor II is true. T.
(e) if both Conclusions I and II are true. 30. How many person are sitting in a row?

3 Adda247 Publications For any detail, mail us at


Publications@adda247.com
50+ Bank PO | Clerk Previous Year’s Papers 2016 – 2020

Statements: (a)
I. A is on 16th position from the right end of the row. M
is on 8th position from the left end of the row. (b)
II. A is 18th from the left end and 20th from the right end (c)
of the row.
(d)
Directions: (31-35): Study the following information
carefully and answer the questions given below: (e)
Six girls – A, B, C, D, E and F are sitting in a row facing North
and six boys – P, Q, R, S, T and U are sitting in a different 37. What will be the code for ?
row facing South, but not necessarily in the same order. (a) 19 (b) 20 (c) 22
Each girl in a row is facing exactly one boy from the other (d) 27 (e) 25
row. The following information is known about them. Directions (38-40) : Study the following information
P sits second to the right of the person who sits opposite B. carefully and answer the questions which follow–
B does not sit at any end of the row. C and D are immediate ‘P – Q’ means ‘P is father of Q’
neighbors of B, and one of them sits at an extremes. E sits ‘P ÷ Q’ means ‘P is sister of Q’
second to the left of D but not opposite R. ‘P × Q’ means ‘P is mother of Q’
P is a neighbor of R but is not opposite F. Neither Q nor T ‘P + Q’ means ‘P is brother of Q’
are adjacent to P. T is adjacent to either R or Q but not both.
U is a neighbor of the person who sits opposite E. 38. Which of the following means ‘A is nephew of B’?
(a) A + C – B × K (b) B ÷ H – A + D
31. Who sits opposite C? (c) B ÷ G – A ÷ R (d) B + T × A ÷ E
(a) S (b) P (c) R (e) None of these
(d) Q (e) T
39. Which of the following means ‘P is grandfather of J’?
32. Who sits second to the left of R? (a) J ÷ W – U – P (b) P × G + J ÷ A
(a) S (b) T (c) P (c) P – B ÷ J ÷ R (d) P – T – J ÷ S
(d) Q (e) U (e) None of these
33. If S and D interchange their positions, then who sits to 40. How is R related to B in the expression ‘B ÷ C – S + R’?
the immediate left of D? (a) nephew or niece (b) niece
(a) U (b) R (c) P (c) nephew (d) None of these
(d) T (e) Q (e) Cannot be determined
34. Four of the following five are alike in a certain way and Directions (41 – 45): Answer the questions on the basis of
so form a group. Find the one which does not belong to the information given below.
that group? A number arrangement machine when given an input of
(a) U, A (b) P, B (c) D, S words, rearranges them following a particular rule in each
(d) C, R (e) D, T step. The following is an illustration of input and steps of
35. Which of the following is true, according to the given rearrangement.
information? Input : 98 Bite 102 Legal 88 54 Mango 21 Cool Zing.
(a) U and C are at the extremes. Step1: Bite 98 Legal 88 54 Mango 21 Cool Zing 102
(b) R sits second to the right of S. Step2: Cool Bite Legal 88 54 Mango 21 Zing 102 98
(c) T sits to the immediate right of Q. Step3: Legal Cool Bite 54 Mango 21 Zing102 98 88
(d) B sits second to the left of A. Step4: Mango Legal Cool Bite 21 Zing 102 98 88 54
(e) All the above Step5: Zing Mango Legal Cool Bite102 98 88 54 21
This is the final arrangement and step 5 is the last step for
Directions (36-37) : Triangle represents (1) and circle this input. As per the rules followed in the above steps, find
represents (0). If triangle appears in unit's place then its out in each of the following questions the appropriate steps
value is 1. If it appears in 10's place its value is doubled to for the given input.
2 like that it continues. Questions based on this Input for the questions.
For example: Input: Kite 19 54 Give 31 Right 72 87 Dream Ace.
41. In step-3, what is position of ‘Ace’ from the left end?
(a) Third (b) Fifth (c) Second
(d) Sixth (e) First
36. How will you represent ‘13’ in this code language?

4 Adda247 Publications For any detail, mail us at


Publications@adda247.com
50+ Bank PO | Clerk Previous Year’s Papers 2016 – 2020

42. In step-5, how many Letters/Numbers are between lighter than only S. T is heavier than V. S is taller than V and
Dream and 72? Q is not the tallest.
(a) Five (b) Two (c) Four
46. How many of them are heavier than T?
(d) Six (e) Three
(a) One (b) Two
43. ‘Kite Give Dream Ace Right 87 72 54 31 19’ in which of (c) Three (d) Five
the following step? (e) None of these
(a) Step-2 (b) Step-4 (c) Step-5
47. How many of them are shorter than Q?
(d) There is no such Step
(a) Two (b) Four
(e) Step-3
(c) Three (d) Five
44. In Step-2, which of the following letter/number is 5th (e) None of these
right of Kite?
48. Who among them is the tallest?
(a) 31 (b)Give (c)Right
(a) V (b) P
(d) 87 (e) 54
(c) T (d) R
45. How many steps would be needed to complete the (e) None of these
arrangement?
49. Who, among them is third from top if arranged in
(a) Four (b) Two (c) Seven
descending order of height?
(d) Six (e) Five
(a) Q (b) V
Directions (46-50): Study the following information (c) S (d) Data inadequate
carefully and answer the questions given below: (e) None of these
P, Q, R, S, T and V are six students studying in a class. Each
50. Who among them is the lightest?
of them has a different height and weight. The tallest is not
(a) V (b) T
the heaviest. T is taller than only P but lighter than R. Q is
(c) P (d) R
taller than S and P and heavier than only T and V. P is
(e) None of these

QUANTITATIVE APTITUDE

Directions (51-55): What will come in place of the question


57. A rectangular grassy plot is 112 m by 78 m. It has a
mark (?) in the following number series?
gravel path 2.5 m wide all round it on the inside. Find
51. 7 151 223 259 277 ? the cost of area of the path which has the cost of
(a) 268 (b) 295 (c) 286 constructing it at Rs. 2 per square metre?
(d) 259 (e) None of these (a) Rs. 1500 (b) Rs. 1600 (c) Rs. 1750
(d) Rs. 1850 (e) None of these
52. 27 30.2 23.8 33.4 20.6 ?
(a) 30.2 (b) 36.6 (c) 39.8 58. Find length of BC ? ∠ CDB= ∠ DAB=90∘
(d) 17.4 (e) None of these
53. 5 4 6 15 56 ?
(a) 280 (b) 275 (c) 270
(d) 265 (e) 285
54. 7 10 21 52 121 ?
(a) 256 (b) 270 (c) 254
(d) 252 (e) None of these ∘
(a) 24 (b) 26 (c) 22
55. 5 3 4 ? 38 (d) 28 (e) 27
(a) 8.5 (b) 6 (c) 7.5
(d) 8 (e) 10 59. Circumference of circle is 44 meter. Find the area of
triangle ?(in 𝑚2) (Given- one angle of triangle is 45∘ )
56. Distance between A and B is 24 km a boat travels from
A to B and comes back in 6 hour. The speed of boat in
still water is thrice the speed of stream. Find the speed
of boat.
(a) 3 km/hr (b) 11 km/hr (c) 7 km/hr
(d) 12 km/hr (e) 9 km/hr
5 Adda247 Publications For any detail, mail us at
Publications@adda247.com
50+ Bank PO | Clerk Previous Year’s Papers 2016 – 2020

(a) 49 (b) 49√2 (c) 98√2 68. Which bank has the lowest number of employees?
(d) 98 (e) None of these (a) SBI (b) PNB (c) OBC
(d) PNB and OBC (e) None of these
60. Find the area of shaded portion ?
69. What is the respective ratio between the total salary of
Rs. 15000 salaried employees to the total salary of Rs.
35000 salaried employees?
(a) 1 : 3 (b) 2 : 3 (c) 1 : 1
(d) 4 : 1 (e) 2 : 7
(a) 154 (b) 196 (c) 156 70. The number of employees of OBC drawing salary Rs.
(d) 198 (e) None of these 20000 is approximately what percent of the number of
employees of SBI drawing salary Rs. 30000 month?
61. A bag contains 4 red, 5 yellow and 6 green balls. 3 balls 1 2
are drawn randomly. (a) 75% (b) 33 2 (c) 16 3
2
What is the probability that the balls drawn contain no (d) 80% (e) 66 3%
yellow ball?
24 33 12 71. Total salary of Rs. 20000 salaried employees is what
(a) (b) (c) percent of the total salary of 25000 salaried
91 91 65
17 employees?
(d) 182 (e) None of these
(a) 92 (b) 90 (c) 94
Directions (62 – 66) : In each of these questions, two (d) 86 (e) None of these
equations (I) and (II) are given. You have to solve both the Directions (72-76): Study the table carefully to answer the
equations and give answer. questions that follow.
(a) if x > y (b) if x  y (c) if x < y Number of animals in grasslands of four different countries
(d) if x  y in five different years
(e) if x = y or no relation can be established between ‘x’ and
y.
62. I. 4x + 7y = 42 II. 3x – 11y = – l
63. I. 9x2 – 29x + 22 = 0 II. y2 – 7y + 12 = 0
64. I. 3x2 – 4x – 32 = 0 II. 2y2 – 17y + 36 = 0
65. I. 3x2 – 19x – 14 = 0 II. 2y2 + 5y + 3 = 0
66. I. x2 + 14x + 49 = 0 II. y2 + 9y = 0 72. What is the average of the number of tigers in the
grassland of Sri Lanka over all the years together?
Directions (67-71): Read the following graph carefully and
(a) 386 (b) 389 (c) 369
answer the questions given below. (d) 276 (e) None of these
Number of employees and their salaries (in thousands per
month) in different banks. 73. What is the difference between the total number of
lions and bears in the grassland of England in the year
2005 and the number of tigers in the grassland of
South Africa in the year 1995?
(a) 597 (b) 558 (c) 677
(d) 668 (e) None of these
74. The total number of animals together in the grassland
of China in the year 1990 is approximately what per
cent of the total number of bears in the grassland of Sri
Lanka over all the years together?
(a) 44% (b) 56% (c) 41%
(d) 47% (e) 51%
75. If 35 per cent of the total number of animals in the
grassland of China in the year 2010 died due to an
67. What is the average salary of employees of PNB? epidemic, how many animals remained in the
(a) Rs. 26125 (b) Rs. 24525 grassland of China in the year 2010?
(c) Rs. 23186 (d) Rs. 25625 (a) 976 (b) 952 (c) 986
(e) None of these (d) 962 (e) None of these

6 Adda247 Publications For any detail, mail us at


Publications@adda247.com
50+ Bank PO | Clerk Previous Year’s Papers 2016 – 2020

76. What is three-fourths of the total number of lions in the 84. The ages of Ranjana and Rakhi are in the ratio of 15 :
grasslands of all the four countries in the year 2000? 17 respectively. After 6 years, the ratio of their ages
(a) 848 (b) 868 (c) 804 will be 9 : 10. What will be the age of Ranjana after 6
(d) 824 (e) None of these years?
(a) 40 years (b) 30 years (c) 34 years
77. The salaries of A, B and C are in the ratio 1 : 3 : 4. If the (d) 36 years (e) None of these
salaries are increased by 5%, 10% and 15%
respectively, then the increased salaries will be in the 85. The simple interest accrued on an amount of Rs.
ratio 20000 at the end of 3 years is Rs. 7200. What would be
(a) 20 : 66 : 95 (b) 21 : 66 : 95 the compound interest accrued on the same amount at
(c) 21 : 66 : 92 (d) 19 : 66 : 92 the same rate in the same period?
(e) None of these (a) Rs. 8342.36 (b) Rs. 8098.56
(c) Rs. 8246.16 (d) Rs. 8112.86
78. Two pipes A and B can separately fill a cistern in 60 (e) None of these
minutes and 75 minutes respectively. There is a third
86. A pipe can empty a tank in 40 minutes. A second pipe
pipe in the bottom of the cistern to empty it. If all the
with diameter twice as that of the first is also attached
three pipes are simultaneously opened, then the
with the tank to empty it. The two together can empty
cistern is full in 50 minutes. In how much time, the
the tank in ;
third pipe alone can empty the cistern? 1
(a) 90minutes (b) 100 minutes (c)110 minutes (a) 8 minutes (b) 133minutes
(d) 120 minutes (e) None of these (c) 30 minutes (d) 38 minutes
(e) None of these
Directions (79-83): Each question below is followed by two
statements I and II. You have to determine whether the 87. A shopkeeper purchased a TV for Rs. 2,000 and a radio
data given in the statement is sufficient to answer the for Rs. 750. He sells the TV at a profit of 20% and the
question. You should use the data and your knowledge of radio at a loss of 5%. The total loss or gain is
Mathematics to choose between the possible answers. Give (a) Gain Rs. 352.50 (b) Gain Rs. 362.50
answer— (c) Loss Rs. 332 (d) Loss Rs. 300
(a) If the question can be answered by using statement I (e) None of these
alone but cannot be answered by statement II alone. 88. 8 men and 4 women together can complete a piece of
(b) If the question can be answered by using statement II work in 6 days. Work done by a man in one day is
alone but cannot be answered by statement I alone. double the work done by a woman in one day. If 8 men
(c) If both statements I and II together are required to and 4 women started working and after 2 days, 4 men
answer the question. left and 4 new women joined. In how many more days
(d) If the answer can be found by using any of the two will the work be completed?
statements alone. (a) 5 days (b) 8 days (c) 6 days
(e) If both the statements together are not sufficient to (d) 4 days (e) 9 days
answer the question. 89. A, B and C started a business with their investments in
79. What will be the cost of painting a rectangular wall? the ratio 1 : 2 : 4. After 6 month A invested the half
I. Cost of painting is Rs 10 per square meter. amount more as before and B invested same the
1
II. Perimeter of wall is 60 m. amount as before while C withdrew 4 th of his
80. Age of A is one third of age of B. What are their ages? investment. Find the ratio of their profits at the end of
I. After 10 years the ratio between age of A and B will the year.
3 : 7. (a) 5 : 12 : 13 (b) 5 : 11 : 14
II. 10 years ago the ratio of their ages was 1 : 5. (c) 5 : 12 : 14 (d) 5 : 12 : 10
(e) None of these
81. What is the length of a train?
90. Richa’s science test consist of 85 questions from three
I. It takes 8 seconds to cross a man.
sections- i.e. A, B and C. 10 questions from section A, 30
II. It takes 20 seconds to cross a 50-meter-long bridge
questions from section B and 45 question from section
with the same speed. C. Although, she answered 70% of section A, 50% of
82. What is sum of two numbers a and b? section B and 60% of section C correctly. She did not
I. a – z = 20 pass the test because she got less than 60% of the total
II. z – b = 20 marks. How many more questions she would have to
answer correctly to earn 60% of the marks which is
83. What is the age of A and B? passing grade?
I. Age of A is 80% of the age of C. (a) 4 (b) 2 (c) 5
II. Age of B is 60% of the age of C (d) 6 (e) 8

7 Adda247 Publications For any detail, mail us at


Publications@adda247.com
50+ Bank PO | Clerk Previous Year’s Papers 2016 – 2020

91. The average age of 28 men is 27 years. If the age of one 96. 15 litres of a mixture contains alcohol and water in the
more man is added to it, the average increases by 1 ratio 1 : 4. If 3 litres of water is mixed in it, the
year. What is the age of the new man? percentage of alcohol in the new mixture will be
2
(a) 28 years (b) 42 years (a) 15 (b) 16 (c) 17
3
(c) 56 years (d) 54 years 1
(e) None of these (d) 18 (e) None of these
2

92. Ms Deepti Jain invests 11% of her monthly salary, i.e., 97. Two pipes A and B can fill a cistern in 30 minutes and
Rs. 5236 in Fixed Deposits. Later she invests 19% of 45 minutes respectively. Both pipes are opened. The
her monthly salary on Life Insurance Policies, also she cistern will be filled in just 20 min, if the pipe B is
invests another 7% of her monthly salary on Mutual turned off after
Funds. What is the total annual amount invested by Ms (a). 5 min (b). 9 min (c). 10 min
Deepti Jain? (d). 15 min (e). None of these
(a) Rs. 21134 (b) Rs. 17612 (c) Rs. 10567 98. 2 men and 3 boys can do a piece of work in 10 days
(d) Rs. 35224 (e) None of these while 3 men and 2 boys can do the same piece of work
93. A committee of 3 members is to be selected out of 3 in 8 days. In how many days can 2 men and 1 boy do
men and 2 women. What is the probability that the the same piece of work?
1
committee has atleast one woman? (a) 12 2 days (b) 24 days (c) 32 days
1 9 9
(a) (b) (c) 1
(d) 11 days (e) None of these
10 20 10 2
1
(d) 20 (e) None of these
99. A man sets out on cycle from Delhi to Faridabad, and at
94. A and B together can complete a piece of work in 12 the same time another man starts from Faridabad on
days. A alone can complete in 20 days. If B does the cycle for Delhi. After passing each other they complete
6 3
work only half a day daily, then in how many days A their journeys in 2 7 and 5 5 hours respectively. At what
and B together will complete the work? rate does the second man cycle if the first man cycles
(a) 10 days (b) 20 days (c) 11 days at 14 kmph?
(d) 15 days (e) None of these (a) 10 kmph (b) 5 kmph (c) 7 kmph
(d) 8 kmph (e) None of these
95. A train is moving at a speed of 132 km/hour. If the
100. The length of rectangular floor is twice its breadth. If
length of the train is 110 metres, how long will it take Rs 256 is required to paint the floor at the rate Rs 2 per
to cross a railway platform 165 metres long? sq m, then what would be the length of floor?
(a) 5 second (b) 7.5 second (c) 10 second (a) 16 m (b) 8 m (c) 12 m
(d) 15 second (e) 9 second (d) 32 m (e) 20 m

ENGLISH LANGUAGE

Directions (101–108): Read the following passage carefully often sceptical. It is bound by time and space. We use our
and answers the questions given below it. Certain words are lower mind to make sense of our complicated and emotional
given in bold to help you locate them while answering some world. The lower mind is the stuff of business schools,
of the questions "operations-focused" education and experiential learning.
There is no fool so great a fool as a knowing fool. But to The lower mind delivers reductionist thinking and
know how to use knowledge is to have wisdom." Businesses mechanistic, conventional approaches to life. The main
continue to face challenging times. In this uncertainty, some drawback of living in the lower mind is that it only reflects
leaders have lost their way due to egregious moral and your internal map of reality. It is like being stuck in your
ethical missteps. Others have reached career dead ends due own intellectual zip code, never moving beyond your nine-
to their inability to see the big picture from a higher not-so- digit thoughts, beliefs, assumptions, expectations and world
common perspective. Many of these leaders are views. It is like living in one town, knowing it completely,
undoubtedly intelligent. But they're not wise. and never venturing outside the borders of that town.
Our minds work on a lower and higher level. The lower level Intelligent people are generally engaged with their lower
deals with the concrete - our immediate physical mind and left-brain thinking. The lower mind focuses on one
environment, information, facts and logic. Our lower mind corner of the painting. Wisdom does not arise from this
supports us to be aware, conceptual and reflective. Our place. The higher mind considers the abstract. It involves
lower mind is rational, analytical, opinionated, busy and intuition, aspiration, heart, soul and spirit and connects
8 Adda247 Publications For any detail, mail us at
Publications@adda247.com
50+ Bank PO | Clerk Previous Year’s Papers 2016 – 2020

Mock SBI Clerk Mains 2018


13
REASONING ABILITY

Directions (1-5): Study the following information carefully Directions (6-10): Study the following information
and wer the questions given below. carefully to wer the given questions.
There are eight members i.e. A, B, C, D, E, F, G and H are A number arrangement machine when given an input line
sitting around a square table such that four of them likes of numbers rearranges them following a particular rule in
flowers i.e. Lily, Rose, orchid and Sunflower and four of each step. The following is an illustration of input and
them likes fruits i.e. Mango, Kiwi, Apple, Banana but not rearrangement.
necessarily in the same order. Those who likes Fruits sits Input: 95 11 76 21 89 42 64 31
at the corner and those who like flower sits at the middle Step I: 02 76 21 89 42 64 31 04
of the table. Some of them face inside and some of them Step II: 03 02 76 42 64 31 04 01
face outside. Step III: 04 03 02 42 64 04 01 01
The one who likes Orchid sits third to the right of H. A sit Step IV: 06 04 03 02 04 01 01 02
second to the right of the one who likes Orchid. A is not the Step IV, is the last step.
immediate neighbour of H. The one who likes Banana is an Input: 75 12 10 94 84 32 63 42 54 22
immediate neighbour of A. The one who likes Banana sits
opposite to the one who likes Kiwi. H does not like Kiwi. 6. How many steps are required to complete the given
The one who likes Apple sits second to the left of the one arrangement?
who likes Kiwi, who is not the immediate neighbour of A. B (a) III (b) V (c) IV
sits third to the right of the one who likes Apple. F likes Lily. (d) VI (e) None of these
Only one person sits between B and the one who likes 7. How many elements are there between ‘03’ and ‘42’ in
Sunflower. F faces the one who likes Sunflower. E and G sits step-II?
opposite to each other. E does not like Banana. C sits (a) Two (b) One (c) Three
second to the right of G. C does not face Inside. C and D face (d) Four (e) None of these
same direction as G.
8. Which number would be at the fourth position from
1. A like which of the following item? the left end in the last step of the output?
(a) Mango (b) Rose (c) Kiwi (a) 04 (b) 05 (c) 64
(d) Sunflower (e) None of these (d) 02 (e) None of these
2. Who among the following sits second to the right of C? 9. What is the sum of the third element from the left in
(a) The one who likes Kiwi step II and 2nd from the right in the last step?
(b) D (a) 78 (b) 76 (c) 65
(c) The one who likes Banana (d) 83 (e) None of these
(d) The one who likes Lily
(e) Both a and b 10. What is the difference between the third element from
the right in step IV and the fourth element from the left
3. Which of the following is not true regarding F? in step III?
(a) F faces the one who likes Sunflower. (a) 21 (b) 24 (c) 28
(b) F and G faces opposite direction. (d) 19 (e) None of these
(c) F sits second to the right of the who likes Orchid.
(d) F and E are immediate neighbours. Directions (11-15): Study the following information
(e) All are correct carefully and wer the questions given below.
Six Horses i.e. A, B, C, D, E, F are standing in a row facing
4. How many persons sit between A and B when counted north at a distance which is a successive multiple of 4 in an
from the left of A?
increasing order from the left. Horse F is second to the right
(a) Three (b) One (c) Two
of Horse C. The total distance between Horse E and D is
(d) Four (e) None of these
52m. Only one Horse stands in between Horse B and Horse
5. Who among the following sits third to the left of D? E. Horse F and Horse D are Standing next to each other.
(a) H (b) C (c) G Horse A and Horse F are not standing next to Horse B. Now
(d) B (e) None of these Horse B starts moving towards north direction after

2 Adda247 Publications For any detail, mail us at


Publications@adda247.com
50+ Bank PO | Clerk Previous Year’s Papers 2016 – 2020

moving 10m its takes a right turn and stops at point T after 17. What will possibly be the code for ‘EMBARKS’?
moving 28m. Horse A starts moving in east direction and (a) **@##1**@#12
after going 12m it turns right and move 20m and then (b) **@##1**@#31
again turn right and move 72m and stops there at point H. (c) **@##1**@#41
Horse C starts moving in south direction and after moving (d) **@##1**@#11
10m it takes a left turn and moves 20m then it again takes (e) None of these
a left turn and moves 5m. From there it takes a right turn 18. What will possibly be the code for ‘SMITTLE’?
and moves 24m and stops at point V. (a) 1#1**@#2#1**
11. What is the shortest distance between Point V and (b) 1#1**@#2#11*
point H? (c) 1#1**@#2#1*1
(a) 10m (b) 15m (c) 5m (d) 2#1**@#2#1**
(d) 20m (e) None of these (e) None of these
12. In which direction and at what distance is point H with 19. What will possibly be the code of ‘ANNUAL’?
respect to F’s initial position? (a) **@##1$$**@#
(a) 20m south (b) 20m, North (b) *1@##1$$**@#
(c) 2m, Southeast (d) 20m, Northwest (c) **##@1$$**@#
(e) None of these (d) *2@##1$$**@#
(e) None of these
13. If horse D, moves 5m in the south direction and reaches
point K then point V is in which direction with respect 20. What is the code for ‘PROM’?
to point K? (a) 5#1$$@# (b) 2#1$$@#
(a) Southeast (b) North (c) East (c) 4#1$$@# (d) 6#1$$@#
(d) West (e) None of these (e) None of these
14. What is the total distance between Horse E and Horse Directions (21-25): Study the following information
A? carefully and wer the questions:
(a) 74m (b) 80m (c) 84m There are three compartments A, B, C such as compartment
(d) 88m (e) None of these A is in west of compartment B and compartment A and
compartment B is in west of compartment C. Twelve boxes
15. In which direction is point T with respect to point H? P, Q, R, S, V, X, Y, Z, K, L, M, N are placed in three different
(a) North (b) Southwest (c) South compartments such as four boxes are placed in each
(d) East (e) Northwest compartment . And these four boxes are placed one above
Direction (16-20): Study the following information and another. Only one box is placed in between V and Z in
wer the given questions: compartment B. R is placed on top in compartment A. Box
In alphabetical series each consonant is assigned a Y is in the immediate west of L. Box L is placed between box
different number from 1-7 (for ex- B is coded as 1, C- M and N. M is placed above N. Two boxes are placed
2……….J-7)and again those numbers get repeated(for ex- between R and S in same compartment. Box Q is placed
K-1, L-2…….so on). immediately above P in the same compartment. Box X is
Besides the above information, following operations are to placed above box V and Z in the same compartment. Box V
be applied for coding the words given in the questions is placed above box Z.
below. These boxes are shifting in other compartments as per the
Each letters of the given questions will be coded as per the cards drawn and only two cards drawn at a time -
given conditions: I. If both the card drawn is heart then the box placed at
I. Vowels appearing before ‘M’ in the Alphabetical series the top in compartment B will be interchanged with
will be coded as ‘**’. the box placed at the bottom of Compartment C.
II. Vowels appearing after ‘M’ in the alphabetical series II. If among the card drawn one is diamond and another
will be coded as ‘$$’. is spade then the box which is second from the bottom
III. Number preceded by vowel will be coded as ‘#1’. in Compartment A will be inter changed with box
IV. No. followed by vowel will be coded as ‘@#’. placed at second from the top in Compartment C.
III. If among the two cards drawn one is Club and another
16. What will possibly be the code for ‘NORMAL’? is Heart then the box placed at top and the box placed
(a) ‘#1$$@##2**@# at the bottom will be interchanged in compartment B.
(b) ‘#1$$@##1**@# IV. If among the two cards drawn one is club and another
(c) ‘#3$$@##1**@# is diamond then the box which is third from the bottom
(d) ‘#1$#@##1**@# in compartment B is interchanged with the box which
(e) None of these is third from the top in compartment A.

3 Adda247 Publications For any detail, mail us at


Publications@adda247.com
50+ Bank PO | Clerk Previous Year’s Papers 2016 – 2020

The Cards Drawn are--- Step 5: If an even number is followed by another even
1. Club-Heart number then the resultant will be the division of first
2. Heart-Heart number by the second number.
3. Club-Diamond 27. Find the sum of two rows
4. Spade-Diamond 8 4 1
Note- The cards will be drawn in the given serial order. 11 6 7
(a) 78 (b) 52 (c) 64
21. Which of the following box is placed at the bottom of
Compartment C after the rearrangement? (d) 76 (e) None of the above
(a) Box K (b) Box L (c) Box M 28. If the sum of the resultants of two rows is 46. Then find
(d) Box Z the value of X.
(e) None of these 9 2 7
22. Which of the following is to the west of box P after the 24 4 X
rearrangement? (a) 16 (b) 27 (c) 8
(a) Box Q (b) Box R (c) Box M (d) 15 (e) None of the above
(d) Box S (e) None of these
29. Find the difference between the resultant of first and
23. Which of the following box is kept at the top in second row.
compartment B after the rearrangement? 13 3 7
(a) Box X (b) Box N (c) Box Y 4 11 12
(d) Box V (e) None of these
(a) 117 (b) 126 (c) 157
24. How many boxes are below box V in its respective (d) 96 (e)None of the above
compartment after the rearrangement?
(a) Three (b) Two (c) One 30. Find the multiplication of the resultant of first and
(d) None (e) None of these second row.
21 19 8
25. Which of the following combination of ‘Box- 16 13 9
compartment’ before the rearrangement is correct?
(a) 110 (b) 85 (c) 100
(a) Box R- C (b) Box N- A (c) Box V- C
(d) 120 (e)None of the above
(d) Box N- C (e) All are correct
26. Statement- Indian Metrological Department has Directions (31-34): Study the following information
forecasted that there will be a situation of flood in City carefully and wer the questions.
A within the next 15 days. Six subjects Hindi, English, chemistry, Mathematics,
Course of Action- I. The population of city A should Physics and Biology are taught at a coaching institute. Each
move to city B within next 15 days to get rid of the subject was given a definite and continuous time slots. (i.e.
problem. there is no gap between two consecutive subjects). In a day
II. The people of city A are advised to preserve a total 14 hours class was scheduled. A subject can start its
necessary items for future consumption. slot from a whole hour or half hour only (i.e. a subject can
(a) Only II (b) Both I and II start from 4 pm, 4 : 30 pm but it cannot starts from 4:13
(c) Only I (d) Neither I nor II pm, 4:03 pm) Hindi class takes place from 11 am to 12:30
(e) None of these pm. Only one class took place between Hindi and
Direction (27-30): There are two rows given and to find out Mathematics. Time slot of Mathematics is twice the time
the resultant of a particular row we need to follow the slot of Hindi. Physics classes starts from 5:30 pm.
following steps: - Chemistry class took place between Hindi and Physics, but
Step 1: If an even number is followed by an odd number it did not take place immediately after or before Physics.
then the resultant will be the addition of both the numbers. Total slot time of English and Biology class is 4.5 hours.
Step 2: If an odd number is followed by a perfect square Number of classes that took place between Biology and
then the resultant will be the difference of that square Physics is same as the number of classes between
number and the odd number.
chemistry and Biology. Time slot of Chemistry is 1 hour
Step 3: If an odd number is followed by another odd
less than the time slot of Mathematics.
number (but not a perfect square) then the resultant will
be the addition of both the numbers. 31. How many lectures are scheduled between Hindi and
Step 4: If an odd number is followed by an even number Biology?
(but not a perfect square) then the resultant comes by (a) None (b) One (c) Two
multiplying the numbers. (d) Three (e) More than three

4 Adda247 Publications For any detail, mail us at


Publications@adda247.com
50+ Bank PO | Clerk Previous Year’s Papers 2016 – 2020

32. Which of the following subject’s time slot is scheduled 38. Who among the following go to gym on 17th April?
from 12:30-2:30 pm? (a) K (b) M (c) S
(a) English (b) Chemistry (c) Biology (d) R (e) None of these
(d) Math’s (e) None of these
39. How many persons go to gym between R and S?
33. Which of the following subject is scheduled just after (a) None (b) One (c) Two
English? (d) Three (e) More than three
(a) Hindi (b) Chemistry (c) Biology
(d) Math’s (e) None of these 40. How many persons are heavier than R?
(a) None (b) One (c) Two
34. What is the duration (in hours) of the Physics’ lecture? (d) Three (e) More than three
(a) 2.5 hour (b) 3 hour (c) 3.5 hour
(d) 1.5 hour (e) 2 hour 41. Who among the following go to gym immediately after
K?
Direction (35-37): In the following questions, the symbols (a) P (b) M (c) N
#, &, @, * , $, % and © are used with the following meanings (d) O (e) Q
as illustrated below. Study the following information and
wer the given questions: Directions (42-44): Study the information carefully wers
A@B- A is the child of B. the questions given below.
A©B- A is the parent of B α me either hour hand or minute hand is at 8
A%B- A is father-in-law of B ∞ me either hour hand or minute hand is at 11
A&B- A is brother-in-law of B ® me either hour hand or minute hand is at 6
A$B- A is brother of B £ me either hour hand or minute hand is at 9
A*B- A is wife of B Å me either hour hand or minute hand is at 2
A#B- A is sister-in-law of B µ me either hour hand or minute hand is at 5
35. If X©F$D&Q@H©E&F, then how is F related to H? Note: if two symbols are given than by default second
(a)father (b)Brother-in-law symbol is consider as minute hand and first symbol is
(c)son-in-law (d)Sister considered as hour hand. And all time are considered at
(e)None of these pm.

36. If G*J$K©Y@V&C#G then how J is related to C? 42. If a man leaves from GIP to WOW at ®Å. Usually he
(a) father (b) Brother-in-law takes 20 min to reach WOW, but that time he reaches
(c) daughter-in-law (d) Brother 15 min later, then at what time he will reach WOW?
(e) None of these (a) ∞® (b) ®∞ (c) ®£
(d) ®® (e) None of these
37. If K&L%M*N@O then how is N related to K?
(a)father (b) Uncle (c) Nephew 43. A person takes 40 minutes to reach airport from his
(d)Sister (e)None of these office and he has to catch airplane that is scheduled to
depart at ‘£µ’ so at what time should he leave from his
Directions (38-41): Study the following information
office for the airport to arrive at the airport at 20
carefully and wer the questions given below.
Nine persons K, L, M, N, O, P, Q, R, S are going to gym on minutes earlier?
three dates 8, 17, 25 of three different month January, (a) µÅ (b) ®∞ (c) ŵ
March and April of same year. Also each of them is of (d) αµ (e) None of these
different weight. Only three persons are heavier than S. Q 44. If a man leaves from his home to office at ‘£∞’ and he
is going to gym immediately after N but not in the same takes 2hr to reach his office, at what time he will reach
month as N. Only one person go to gym in between O and R to office?
and all go in the same month. K is just heavier than M but (a) µ£ (b) ∞Å
lighter than Q. L is heavier than S but lighter than N who is (c) ∞∞ (d) Can’t be determined
not the heaviest. Only three persons go to gym in between (e) None of these
L and S but none of them go to gym in a month having 30
days. The one who go immediately after O is not lighter Directions (45-48): Study the following information
than O. The one who is the lightest person attend gym carefully and wer the questions given below:
immediately after L. As many persons go to gym in Ten vehicles are placed in two parallel rows, five vehicles
between N and K as between K and R. P is lighter than both in row 1 facing north and rest in row 2 facing south having
M and O. R is not lighter than K. The one who go to the gym equal distance between each other i.e. each of them faces
last is heavier than O but lighter than S. another vehicle.

5 Adda247 Publications For any detail, mail us at


Publications@adda247.com
50+ Bank PO | Clerk Previous Year’s Papers 2016 – 2020

These vehicles are placed according to speed which (c) (200-250) km/h
increases from West to East direction in both rows but not (d) (250-275) km/h
necessarily in the given order. Two vehicles are placed (e) None of these
between boat and the vehicle with speed 170 km/h. Car is
Directions (49): Each question given below consists of a
placed just next to the vehicle which faces Truck. Only one
statement, followed by two arguments numbered I and II.
vehicle is placed between Car and cycle. Scooter is just next
You have to decide which of the arguments is a ‘strong’
to the vehicle with speed 120 km/h, but none of them is
argument and which is a ‘weak’ argument. Give wer-
placed at extreme end of the row. All the vehicle with speed
(a) If only argument I is strong
less than 50 km/h faces South. Aeroplane is placed next to
(b) If only argument II is strong
vehicle with speed 228 km/h.
(c) If either I or II is strong
(d) If neither I nor II is strong
(e) If both I and II are strong
49. Statement: Should all the unauthorized structures in
the city be demolished?
Arguments:
I. No. Where will the people residing in such houses
live?
II. Yes. This will give a clear message to general public
and they will refrain from constructing
unauthorized buildings.
45. Which among the following vehicle is placed opposite Directions (50): In question below is given a statement
to Motor Cycle? followed by two assumptions numbered I and II. An
(a) Auto rickshaw (b) Train assumption is something supposed or taken for granted.
(c) Cycle (d) Car You have to consider the statement and the following
(e) Cannot be determined assumptions and decide which of the assumptions is
implicit in the statement.
46. What is the speed of the vehicle which is placed at Give wer-
extreme left end of row 1? (a) If only assumption I is implicit
(a) 54 km/h (b) 12km/h (b) If only assumption II is implicit
(c) 290 km/h (d) 274 km/h (c) If either I or II is implicit
(e) 22 km/h (d) If neither I nor II is implicit
47. Which vehicle is placed at extreme left end of row 2? (e) If both I and II are implicit
(a) Train (b) cycle (c) Helicopter 50. Statement: “I would like to study the impact of pay
(d) Aeroplane (e) Scooter revision on job satisfaction of employees.” — A tells B.
48. What is the speed range of the vehicle which is placed Assumptions:
exactly between Car and Aeroplane? I. Job satisfaction can be measured.
(a) (125-160) km/h II. A has necessary competence to undertake such
(b) (160-200) km/h study.

QUANTITATIVE APTITUDE

Directions (51-56): - Rahul goes to gym and runs 40 together can complete same work in 3 hours. Rahul is (C)%
minutes on treadmill. For starting 15 minutes he runs at a more efficient than P1. After that work he comes back to
uniform speed of 5 km/hr and after that he runs at a home in upstream (Speed of stream is 3km/hr and his
uniform speed of 9km/hr for remaining time. He runs total speed in still water and distance between his house and
(A) km on treadmill. After that he comes to his house and office are same as earlier). He takes (D) hours to reach
get ready for office which is 45km away from his house. He home.
reaches office in 1.5 hours at 9:30 a.m. When he reaches home, two of his friends Aman and
In office he gives some work to his subordinates P1 and P2 Raman come at his house. All three starts to play a game in
at (B). P1 can complete that work in 6 hours while which 2 dices are used by each person. (E) is the number
efficiency of P1 and P2 is in the ratio 5 : 4. P1 and P2 together of outcomes in which first Rahul and then Aman throw
completes 75% of that work at 12:30 p.m. Rahul and P2 their respective dices. In a game, all three throw their dices

6 Adda247 Publications For any detail, mail us at


Publications@adda247.com
50+ Bank PO | Clerk Previous Year’s Papers 2016 – 2020

and each one of them get 8 as the sum of numbers in their original rectangle is increased by 6 cm. Find the
dices and any one of two not get same outcomes. Winner is perimeter of the rectangle.
the one who gets highest number as the sum of the square (a) 42 cm (b) 88 cm (c) 80 cm
of the number comes in dices. (F) should be the outcomes (d) 84 cm (e) 72 cm
of the dices of Raman if Raman is winner of the game.
58. Breadth of a rectangle is equal to the diagonal of the
51. What value will come at the place of ‘A’? square whose side is 2.5√2 cm. Ratio between length
(a) 4.25 km (b) 3.75 km (c) 5 km and breadth of rectangle is 3 : 1. Find the area of the
(d) 5.25 km (e) None of the given options rectangle (in cm2).
52. What value will come at the place of ‘B’? (a) 125 (b) 75 (c) 90
(a) 10:45 a.m. (b) None of the given options (d) 100 (e) 115
(c) 11 a.m. (d) 10:30 a.m. 59. Equal distance is covered by a boat in upstream and in
(e) 10 a.m. downstream in total 5 hours. Sum of speed of a boat in
53. What value will come at the place of ‘C’? upstream and downstream is 40 km/hr. Speed of boat
2 in still water is 600% more than the speed of stream.
(a) 16 3 % (b) 20% (c) 25%
Find the approximate distance covered by boat in
1
(d) 333 % (e) 50% downstream (in km).
(a) 40 (b) 35 (c) 55
54. What value will come at the place of ‘D’? (d) 59 (e) 50
7
(a) 2 hours (b) 1.5 hours (c) 1 8 ℎ𝑜𝑢𝑟𝑠
2 4
60. A and B entered into a partnership with Rs.800 and
(d) 1 ℎ𝑜𝑢𝑟𝑠 (e) 1 ℎ𝑜𝑢𝑟𝑠 Rs.1600 respectively. From 9th months on-ward they
3 11
each decided to invest Rs.100 more on starting of each
55. What value will come at the place of ‘E’?
month. If total annual profit is Rs.7700 then find the
(a) 72 (b) 42 (c) 36
profit share of A.
(d) 108 (e) 54
(a) Rs.2650 (b) Rs.3250 (c) Rs.4250
56. What value will come at the place of ‘F’? (d) Rs.2350 (e) Rs.1650
(a) None of the given options
61. A starts a business, after 6 months B also join him with
(b) Cannot be determined
Rs.4500 and after 2 months of B’s joining C also join
(c) 3 and 5
them with Rs.4500. If A gets approx Rs 4900 out of
(d) 4 and 4
total annual profit of Rs.10,000 then find the
(e) 2 and 6
approximate value of initial investment of A.
57. If length of a rectangle is decreased by 6 cm we get a
(a) Rs.4800 (b) Rs.4200 (c) Rs.3600
square and the area of square formed is 252 cm² less
(d) Rs.4400 (e) Rs.5200
than the area of square formed when breadth of the

Directions (62-66): - Bar chart given below shows selling price of five articles and profit % earned on selling these articles
by Ravi. Study the data carefully & wer the following questions.

Selling Price Profit %


160
140
Selling price is in Rs. and

120
Profit % is in %

100
80
60
40
20
0
A B C D E

7 Adda247 Publications For any detail, mail us at


Publications@adda247.com
50+ Bank PO | Clerk Previous Year’s Papers 2016 – 2020

62. Ravi sold article ‘D’ to Shyam who again sold it at 25% 69. 12 men can complete a work in 10 days. 18 women can
profit. Find the difference between profit earned by do the same work in 20 days. 27 children can do that
Ravi to profit earned by Shyam. work in 20 days. 9 women and 9 children together do
(a) Rs. 5 (b) Rs. 10 (c) Rs. 15 that work for 16 days.
(d) Rs. 20 (e) Rs. 25 Quantity I: No. of men required to complete the
remaining work in one day
63. Cost price of article ‘A’ is what percent more/less then
Quantity II: 36
cost price of article ‘C’.
(a) Quantity I > Quantity II
(a) 62.5% (b) 37.5% (c) 25% (b) Quantity I ≤ Quantity II
(d) 75% (e) 50% (c) Quantity I = Quantity II or No relation
64. Ravi marked article B, 50% above its cost price, then (d) Quantity I < Quantity II
what percent discount should be given on marked (e) Quantity I ≥ Quantity II
price to earn the given profit? 70. Quantity I: Time taken to fill the tank when A, B and C
(a) 40% (b) 30% (c) 25% are opened in every alternate minute starting with A
(d) 20% (e) 10% and ending with C. A, B and C alone takes 20 minutes,
65. Profit earned on selling article ‘E’ is how much 15 minutes and 12 minutes respectively to fill the tank.
more/less than profit earned on selling article ‘C’. Quantity II: Find the time taken by waste pipe to empty
(a) Rs.40 the full cistern. Two pipes alone can fill a cistern in 10
(b) None of the given options minutes and 15 minutes respectively. When these two
(c) Rs.30 pipes along with the waste pipe are opened, the cistern
(d) Rs.20 gets filled in 18 minutes.
(e) Rs.10 (a) Quantity I > Quantity II
(b) Quantity I < Quantity II
66. Ravi mark-up article ‘A’ such that on selling article ‘A’ (c) Quantity I ≥ Quantity II
at 16% discount he will earn the given profit. Mark up (d) Quantity I ≤ Quantity II
price of article ‘A’ is what percent more than its cost (e) Quantity I = Quantity II or No relation
price?
1 2 2 Directions (71-75): - Data given below shows number of
(a) 33 3 % (b) 66 3 % (c) 16 3 %
units of electricity consumed by F, Lights and Other
2 1
(d) 26 % (e) 73 % appliances in three different houses. Study the data
3 3
carefully and wer the following questions.
Direction (67-70): - Two quantities that is I and II are given House A → Total number of units consumed in House ‘A’ is
in following questions. Students is expected to solve the 250 units out of which 120 units are consumed by Other
quantities and wer them according to given options by appliances. Units consumed by F is 30 less than Units
comparing their numerical values. consumed by Lights.
67. 3𝑥+5 . 92𝑥−4 = 95𝑥−14 House B → Units consumed by Lights in House ‘A’ and
And, 2𝑦 2 − 15𝑦 − 28 = 3𝑦 2 − 23𝑦 − 13 House ‘B’ is same. Units consumed by F in House ‘B’ are
Quantity I: - Value of ‘x’ 60% more than that of f in House ‘A’.
Quantity II: - Value of ‘y’ House C → Total units consumed by Lights in all three
(a) Quantity I > Quantity II houses is 200 units and units consumed by F and Lights is
(b) Quantity I < Quantity II same in House C. Units consumed by Other appliances is
125% more than that by F in this House. Total units
(c) Quantity I ≥ Quantity II
consumed by Other appliances in all three houses is 320
(d) Quantity I ≤ Quantity II units.
(e) Quantity I = Quantity II or No relation
71. Number of units consumed by Lights in House ‘B’ is
68. Quantity I: When an article sold at 28% discount then what percent more of the units consumed by Lights in
profit earned is 29.6%. ‘x’ is the profit % when article house ‘C’?
sold at 30% discount. (a) 100% (b) 200% (c) 120%
Quantity II: 38 (d) 50% (e) 150%
(a) Quantity I = Quantity II or No relation 72. Average number of units consumed by Other
(b) Quantity I < Quantity II appliances in House ‘B’, ‘C’ and ‘D’ is 110 units. Find the
(c) Quantity I ≤ Quantity II units consumed by Other appliances in House ‘D’?
(a) 110 units (b) None of the given options
(d) Quantity I ≥ Quantity II
(c) 130 units (d) 120 units
(e) Quantity I > Quantity II (e) 140 units
8 Adda247 Publications For any detail, mail us at
Publications@adda247.com
50+ Bank PO | Clerk Previous Year’s Papers 2016 – 2020

73. Find total number of units consumed in House ‘A’ and 78. B and E started a business together. B left the business
‘C’ together? 9 months after starting of business. Find the difference
(a) None of the given options between profit shares of B and E if total annual profit
(b) 410 units is Rs. 15,400?
(c) 430 units (a) Rs.2100 (b) Rs.4200 (c) Rs.1400
(d) 400 units (d) Rs.2800 (e) Rs.3500
(e) 420 units 79. A and D started a business together after 6 months ‘A’
74. Find the difference between Units consumed by Other is replaced by ‘C’. D left the business after 2 months of
appliances in House ‘B’ and house ‘C’? ‘A’ while ‘C’ worked for total ‘x’ months. Out of total
(a) 10 units profit of Rs 13,050, ‘A’ got Rs 6750, then find the value
(b) 20 units of ‘x’.
(c) 30 units (a) 10 (b) 8 (c) 6
(d) None of the given options (d) 4 (e) 2
(e) 40 units 80. ‘A’, ‘F’ and ‘C’ started a business together. F invested Rs.
4000 more than amount invested by C. F left the
75. Total units consumed by F and Lights together in
business after 6 months of starting of business. After 2
House ‘C’ is what percent less than total units
months more, ‘C’ left the business. Out of annual profit
consumed by Lights and Other appliances together in
if A and C together got Rs 8750 then find total annual
House ‘A’?
profit got by all three together?
(a) 20% (b) 40% (c) 50% (a) Rs 11,250 (b) Rs 10,000
(d) 60% (e) 80% (c) Rs 12,500 (d) Rs 13,750
76. Cost price of a pen is 50 Rs. and that of notebook is 140 (e) Rs 15,000
Rs. If pen is sold at 200% profit, then to purchase 10 Directions (81-85): Solve the given quadratic equations
such note books how many pens are required to sell if and mark the correct option based on your wer—
only profit money is used to buy notebooks?
(a) 16 (b) 18 (c) 14 81. I. (x − 2)² = 9
(d) 20 (e) 22 II. (2y + 8)² = 16
(a) x < y
77. Length of two trains are 150 m and 200 m respectively (b) No relation can be established between x and y.
and the ratio (shorter: longer) of their speed is 2 : 5. If (c) x > y
they cross each other in opposite direction in 15 (d) x ≤ y
second then in what time faster train will overtake the (e) x ≥ y
slower train.
82. I. x² − 16x + 64 = 0
(a) 20 seconds (b) 25 seconds
II. y² − 16y + 63 = 0
(c) 32 seconds (d) 35 seconds
(a) x > y
(e) 27 seconds
(b) x ≤ y
Directions (78-80): Pie-chart given below shows (c) x ≥ y
investment (in terms of percentage) out of total (d) x < y
investment of five different persons. Study the questions (e) No relation can be established between x and y.
carefully and wer them. 25 15
83. I. x2 − x + 2 = 0
Total Investment = Rs 80,000
40 13
II. + 1 =
y² y
E, 15% (a) x < y
(b) x ≥ y
A, 25% (c) No relation can be established between x and y.
D, 10% (d) x ≤ y
(e) x > y
48 14
84. I. x² − x + 1 = 0
C, 15%
45 1
II. + = 2
B, 35% y² y
(a) No relation can be established between x and y.
(b) x ≤ y (c) x < y
(d) x > y (e) x ≥ y

9 Adda247 Publications For any detail, mail us at


Publications@adda247.com
50+ Bank PO | Clerk Previous Year’s Papers 2016 – 2020

85. I. x² + 3x – 4 = 0 III. Difference between the C.I. and S.I. earned in two
II. y² + 10y + 24 = 0 years on the same amount and at the same rate of
(a) x ≤ y interest is Rs. 120.
(b) x < y (a) Only I and III
(c) x > y (b) Only III
(d) No relation can be established between x and y. (c) Only II and III
(e) x ≥ y (d) Cannot be wered even including all statement
(e) None of these
Directions (86-90): The following questions are
accompanied by three statements (I), (II), and (III). You 90. What is the sum of two number?
have to determine which statement(s) is/are sufficient I. The bigger no. is 6 more than the smaller no.
/necessary to wer the questions II. 40% of smaller no. is equal to 30% of bigger no.
III. The ratio b/w half of the bigger no. & one-third of
86. What is distance between A and B ? smaller no. is 2 : 1.
I. Two persons Amit and Abhi started (a) Only II & III
simultaneously from A to B with their speed in (b) Only I & II
ratio 4 : 5. (c) Any two of the three statements
II. Abhi reached reached B one hour earlier than (d) All statement is required
Amit. (e) Only I & either II or III
III. Difference between speed of Amit and Abhi is 20
km/hr. 91. A container contains 165 liters of milk. Some quantity
of milk is taken out and half of that quantity of milk,
(a) Only I and II.
water is added in the container. Now ratio of milk to
(b) Only II and III
water in the container becomes 5:3. What is the
(c) All I, II and III
quantity of water added in it?
(d) Cannot be wered even including all three
(a) 40 lit (b) 45 lit (c) 60 lit
statement
(d) 30 lit (e) 90 lit
(e) None of these
92. Two boxes contain 4 and 16 balls respectively. Two
87. What is the area of rectangle ? balls in the first box and four in the second, are black.
I. If ratio of length and breadth of the rectangle is If a box is chosen randomly and two balls are drawn at
3 : 2. random from it, what is the probability that at least one
II. Circumference of a circle is 440 m and breadth of ball is black if the ball is not replaced?
rectangle is 1/7 th of radius of the circle. 11 43 77
(a) 20 (b) 120 (c) 120
III. If length is 50% more than breadth.
9
(a) Only III (d) 20 (e) None of these
(b) Only II and either I or III. 93. Train A, travelling at 84 kmph, overtook train B,
(c) Only II traveling in the same direction, in 10 seconds. If train
(d) All I, II and III B had been traveling at twice its speed, then train A
(e) None of these would have taken 22.5 seconds to overtake it. Find the
88. How many students failed in class 11th ? length of train B, given that it is half the length of train
A.
I. 400 Students passed in class 11th.
(a) 50 m (b) 100 m (c) 200 m
II. No. of students failed in class 11th is 20% of those
(d) 150 m (e) None of these
failed in class 12th.
III. Ratio of student appeared to that of failed in class 94. A solid sphere of radius 6 cm is melted and re-casted
11th is 5 : 3. into a hollow cylinder of uniform thickness. If the
(a) Only I and III external radius of the base of the cylinder is 5 cm and
(b) Only II its height is 32 cm, find the uniform thickness of the
(c) Only I and II cylinder ?
(d) All I, II and III (a) 3 cm (b) 1.5 cm (c) 1 cm
(e) Cannot be obtained even including all three (d) 2.5 cm (e) None of these
statement 95. X and Y entered into partnership with Rs. 700 and Rs.
89. What is the rate of interest? 600 respectively. After another 3 months, X withdrew
I. S.I. accrued in two years on an amount at same rate two-sevenths of his stock but after 3 months, he puts
of interest is Rs. 44,000. back three-fifths of what he had withdrawn. The total
II. The amount after some years on S.I. is Rs. 154000. profit at the end of the year is Rs. 726. How much of
this should X receive?

10 Adda247 Publications For any detail, mail us at


Publications@adda247.com
50+ Bank PO | Clerk Previous Year’s Papers 2016 – 2020
2 2 1
(a) Rs. 336 (b) Rs. 366 (c) Rs. 633 (a) 26 3 % (b) 18 3 % (c) 33 3 %
(d) Rs. 663 (e) None of these 2 1
(d) 16 % (e) 58 %
3 3
Direction (96-100): - Table given below shows percentage
of men out of total men who worked on odd days in three 97. Total man hour of April is how much more or less than
different months and rest of the men are working on even the total man hour of August?
days of the respective month. Study the table carefully and (a) 80,000 (b) 83,200 (c) 84,800
wer the following questions. (d) 86,400 (e) 88,000
Percentage of 98. Find the ratio between man-hour on even days of
Total Number of men worked on march to man hour on even days of August?
Months
Men worked odd number (a) 6 : 1 (b) 7 : 3 (c) 8 : 3
days (d) 3 : 7 (e) 9 : 2
March 1000 30%
April 1500 20% 99. Man-hour on odd days of April is how much % less than
August 750 60% the man hour on odd days of August?
1
Note: Each man works for 8 hours per day (a) 33 % (b) 40% (c) 37.5%
3
Total man-hours = Total man worked × Total day of work (d) 62.5% (e) 60%
× 8 hours 100. What is the average of man hours on even days of all
96. Total man-hours on odd days of march is what % of the three months together.
(a) 88,000 (b) 66,000 (c) 86,000
total man-hours on even days of April?
(d) 78,000 (e) 74,000

ENGLISH LANGUAGE

Directions (101-105): In each of the following questions COLOUMN A


given below, a sentence is given with some bold words (1) Mislead (2) Emissions
which may contain errors. Below each of the sentence, a (3) Consensus (4) Information
table is given with two columns in which column ‘A’ COLOUMN B
contains the list of bold words, and in column ‘B’ the (5) Misconceptions (6) Eminences
suggested corrections are listed. You have to choose the (7) Compliance (8) Informed
best alternate among the four given options. If no (a) (4) – (8) (b) (2) – (6)
correction is required against the given bold words, mark (c) (1) – (5) (d) Both (1) - (5) and (3-7)
(e) .i.e. “None of the above” as your answer. (e) None of the above
101. The Indian telecom sector has witnessed a Paranoid 103. India has told messaging platform WhatsApp to get
shift—crossing the billion-user mark and rolling out serious about hawking the purveyors of fake news and
new technologies along with other leading markets. variance rumours. In a meeting with one of its top
Just four years ago, it faced multiple challenges— hosts, the law minister stressed that over 20 lives had
falling revenues and increasing debt were wreaking been lost in India due to this rumour-mongering.
triumph. COLOUMN A
COLOUMN A (1) Hawking (2) Variance
(1) Paranoid (2) Rolling (3) Hosts (4) Stressed
(3) Faced (4) Triumph COLOUMN B
COLOUMN B (5) Tracking (6) Vicious
(5) Paradigm (6) Rolled (7) Honchos (8) Guessed
(7) Face off (8) Havoc (a) (4) – (8) (b) (2) – (6)
(a) Both (1) - (5) and (4) – (8)
(c) (1) – (5)
(b) (2) – (6)
(d) (1) - (5), (2) – (6) and (3-7)
(c) (1) – (5)
(e) None of the above
(d) (3-7)
(e) None of the above 104. Preserving, changing social structures, raising
102. The government launched Tarang Sanchar portal to disposable incomes, industrial growth and
correct mislead around electromagnetic emissions infrastructure expansion, coupled with the easy
from towers. It allows people to check emission (EMF) availability of housing loans, are the drivers of
consensus status of mobile towers, giving information decorative paints industry, which accounts for 75% of
about any site, on request, via email. the estimated $8.2-billion Indian market.

11 Adda247 Publications For any detail, mail us at


Publications@adda247.com

You might also like